SlideShare ist ein Scribd-Unternehmen logo
1 von 34
MG2351 Principles of Management

Department of ECE

2013 - 2014

UNIT – I
Part A
1. Define Management. (April 2008)
Management is the process of designing and maintaining an environment in which
individuals, working together in groups, accomplish their aims effectively and efficiently.
According to Koontz and Weilhrich "Management is process of designing and maintaining
of an environment in which individuals working together in groups, efficiently and
efficiently attain the organizational goals."
2. State the Functions of Manager. (May 2010)
P – Planning
O – Organising
S – Staffing
D – Directing
CO – Coordinating
R – Reporting
B – Budgeting
3. What are the various skills required by a manager?
Managers require three kinds of skills
Technical Skill – Knowledge of and proficiency in working with tools and
technology
Human Skill – Ability to work with people
Conceptual Skill – Ability to recognize important element in a situation and
understand relationship among elements
4. Write some characteristics of Management
Management is a continuous process. 2) Managers use the resources of the organisation both
physical as well as human to achieve the goals. 3) Management aims at achieving the
organisation goals by ensuring effective use of resources.
5. Define Productivity. (May 2009)
It implies effectiveness and efficiency in individual and organizational performance.
Productivity = Outputs
Inputs
(within a time period, quality considered)
6. What is Effectiveness & Efficiency ? (Dec 2010)
Effectiveness is the achievement of objectives and Efficiency is the achievement of the ends
with the least amount of resources. Effectiveness is doing right things & efficiency is doing
things rightly.
7. What are management levels?
Top level management. 2) Middle level management. 3)Lower level management.
8. Write some important functions of top level management. (May 2007)
• To formulate goals and policies of the company.
• Formulate budgets.
• To appoint top executives.
9. What is social responsibility?
Social responsibility is the part of the management to initiate actions to protect the interest
of the society.
10. Write Fayol's fourteen principles of management.
1) Division of work. 2) Authority and Responsibility.
3) Discipline. 4) Unity of command. 5) Unity of direction.
6) Subordination of Individual interest to general interest.
7) Remuneration. 8) Centralisation 9) Scalar chain.
MG2351 Principles of Management

Department of ECE

2013 - 2014

10) Order. 11) Equity. 12) Stability of Tenure.
13) Initiative. 14) Esprit de Corps.
11. What is scalar chain?
The instructions and orders should be sent from the top management to the lower management.
12. Explain: Management is both —A science and an art.
Management is a science because it contains general principles. It is also an art because it
requires certain personal skills to achieve desired results.
13. What is Esprit-de-corps?
It means ‘Unity is strength’. In an organization, amongst the employees there should be
harmony and unity.
14. List out the claimants of the business enterprise.
1) Shareholders. 2) Employees.
3) Customers. 4) Creditors.
5) Suppliers. 6) Government
15. What are the major contributions of Taylor?
The major contributions of F.W.Taylor are as follows
• He developed the principle of division of labor/work
• He developed method study
• He advocated time study
• He developed certain principles to breakup each job into small independent elements
• He developed the concept of fair day’s work
• He proposed the functional organization
16. Write some important functions of top level management. (May 2009)
To formulate goals and policies of the company. 2) formulate budgets. 3) To appoint top
executives.
17. What is time study?
Time study refers to the measuring of (under controlled conditions) the time for completing a
particular process using specific materials and techniques. This will be useful in arriving at
a scientific standard for each work task in the
marketing process.
18. Define partnership
Under the Indian Partnership Act of 1932, Partnership is defined as –“the relation between
persons who have agreed to share the profits of a business carried on by all or by any one
of them acting for all”.
19. What do you mean by ‘unity of command’? (Dec 2010)
‘Unity of command’ is an important principle proposed by Henry Fayol.
This implies
that an employee should receive orders from one superior
only.
20. What is centralization?
The organization is centralized when the power is concentrated in the hands of few people.
Part B
1. Explain the Functions of Management.
Planning
Planning is the function that determines in advance what should be done. It is looking ahead
and preparing for the future. It is a process of deciding the business objectives and charting
out the methods of attaining those objectives. In other words, it is the determination of what
is to be done, how and where it is to be done, who is to do it and how results are to be
evaluated. This is done not only for the organisation as a whole but for every division,
department or sub-:unit of the organisation. Thus, planning is a function which is performed
by managers at all levels—top, middle and supervisory. Plans made by top management for
MG2351 Principles of Management

Department of ECE

2013 - 2014

the organisation as a whole may cover periods as long as five or ten years. Plans made by
middle or first line managers, cover much shorter periods. Such plans may be for the next
day's work, for example, or for a two-hour meeting to take place in a week.
Organising
To organise a business is to provide it with everything useful to its functioning: personnel,
raw materials, tools, capital. All this may be divided into two main sections, the human
organisation and the material organisation. Once managers have established objectives and
developed plans to achieve them, they must design and develop a human organisation that
will be able to carry out those plans successfully. According to Alien, this organisation
refers to the "structure which results from identifying and grouping work, defining and
delegating responsibility and authority, and establishing relationships."
Staffing may also be considered an important function involved in building the human
organisation. In staffing, the manager attempts to find the right person for each job.
Staffing fixes a manager's responsibility to recruit and to make certain that there is enough
manpower available to fill the various positions needed in the organisation. Staffing involves
the selection and training of future managers and a suitable system of compensation.
Staffing obviously cannot be done once and for all, since people are continually leaving,
getting fired, retiring and dying. Often too, the changes in the organisation create new
positions, and these must be filled.
The other aspect of organizing involves material organization – organizing other inputs like
materials, equipment, facilities, power, information systems, finance, etc. for effective running
of the organization.
Different objectives require different kinds of organisation to achieve them. For example,
an organisation for scientific research will have to be very different from one for
manufacturing bicycles. Producing bicycles requires assembly-lint techniques, whereas
scientific research requires teams of scientists and experts in various disciplines. Such
people cannot be organised on an assembly-line basis.
Directing
After plans have been made and the organisation has been established and staffed, the
next step is to move towards its defined objectives. This function can be called by various
names: 'leading’, 'directing, 'motivating", 'actuating’, and so on. But whatever the name
used to identify it, in carrying out this function the manager explains to his people what
they have to do and helps them do it to the best of their ability. Directing thus involves
three sub-functions—communication, leadership and motivation. Communication is the
process of passing information and understanding from one person to another. Leadership
is the process by which a manager guides and influences the work of his subordinates.
Motivation means arousing desire in the minds of workers to give their best to the
enterprise. It is the act of stimulating or inspiring workers. If the workers of an enterprise
are properly motivated they will pull their weight effectively, give their loyalty to the
enterprise, and carry out their task effectively. Two broad categories of motivation are:
financial and non-financial. Financial motivation takes the form of salary, bonus, profitsharing, etc., while non-financial motivation takes the form of job security, opportunity of
advancement, recognition, praise, etc.
Controlling
The manager must ensure that everything occurs in conformity with the plans adopted,
the instructions issued and the principles established. This is the controlling function of
management, and involves three elements.
1. Establishing standards of performance.
2. Measuring current performance and comparing it against the established standards.
3. Taking action to correct any performance that does not meet those standards.
MG2351 Principles of Management

Department of ECE

2013 - 2014

In the absence of sound control, there is no guarantee that the objectives which have been
set will be realised. The management may go on committing mistakes without knowing
them. Control compels events to conform to plans.
2. Discuss whether Management is a Science or Art.
Professional managers need to adopt a rational, scientific approach in dealing with the concrete,
measurable aspects of business such as setting objectives/goals, formulating action plans,
efficiently organizing required resources, and devising controls. In this respect, management is a
science. On the other hand, managers have also to deal with the human aspects of the
organization – providing leadership, motivation, guidance to individuals and groups, and
resolving conflicts. In this respect, management is an art.
3. Explain the Henry Fayol’s principles of management? (April 2009)
Give brief explanations of Fayol’s 14 principles listed below: 1) Division of work. 2) Authority and
responsibility 3) Discipline 4) Unity of command. 5) Unity of direction. 6) Subordination of individual
interest to general interest. 7) Remuneration. 8) Centralization 9) Scalar chain 10) Order 11)
Equity 12) Stability of Tenure 13) Initiative and 14) Esprit de Corps.
4. Explain the contributions of FW Taylor.
Explain the salient features of Scientific Management, Time and Motion Study, Differential
Payment system, Reorganization of Supervision, Scientific Recruitment and Training of
Workers, and Cooperation between Management and Workers proposed by Taylor.
5. What are the managerial skills required at different levels of management? (May 2013)
Skills needed by managers: To be successful, managers need four kinds of skills, though the
relative importance of the skills differ at various levels of management.
[i] Technical skill: Knowledge, proficiency in activities involving methods, processes,
procedures.
[ii] Human skill: Ability to work with, interact with people.
[iii] Conceptual skill: Ability to see the ‘big picture’, to understand the importance and
relationships among elements.
[iv] Design skill: Ability to solve problems.
6. Explain the importance of management? How Management is differentiated from
administration
While numerous definitions of the term exist in management literature, one convenient definition
that meets our purpose would be: “the process of planning, organizing, directing and controlling
human efforts to achieve organizational objectives”.
Importance: Management is essential in all organized effort, be it a business activity or any other
activity. It is required for all types of organizations – business and non-business, for-profit and notfor-profit, manufacturing and services. Ever since people began forming groups to accomplish aims
they could not achieve as individuals, managing has been essential for coordinating individual
MG2351 Principles of Management

Department of ECE

2013 - 2014

efforts. In a competitive economy, the quality and performance of management determine the
success of an organization; indeed, they determine its very survival.
7. Explain the Weber’s ideal bureaucracy.
•

Specialization of laws

•

Formal rules and procedures

•

Impersonality

•

Well defined hierarchy

•

Career advanced based on merit

8. Environmental factors: (May2013)
•

Political and legal

•

Economy

•

Competition

•

Technology

•

Socio – cultural

•

Natural environment

9. Explain the role of managers in detail. (Nov 2012)
Interpersonal roles:
• Figurehead: Duties of a ceremonial nature such as inaugurating functions.
• Leader: Motivate, encourage, guide employees.
• Liaison: Interact with other departments, divisions, outsiders.
II Informational roles:
• Recipient: Receive information about activities.
• Disseminator: Pass information to subordinates.
• Spokesman: Transmit information to outsiders.
III Decision roles:
•
Entrepreneur: Seek, develop ideas for new products, markets, technology, etc.
•
Disturbance handler: Solving problems.
•
Resource allocator: Allocate work, resources, delegate authority.
•
Negotiator: Negotiate with clients, suppliers, unions, etc.
10. What are the trends and challenges of management?
• Workforce diversity
• Ethics
MG2351 Principles of Management
•
•
•
•
•
•
•

Department of ECE

2013 - 2014

Innovation and change
Total Quality Mgt
Re-engineering
Empowerment and teams
Bimodal workforce
Downsizing
Contingent workers

UNIT – II
Part A
1. Definition of planning. (May 2008)
The determination of what is to be done, how and where it is t be done, who is to do it and
how results are to be evaluated.
2. Types of planning
Short range and long range planning repeated and single use planning and functional
planning
3. What are the Qualities of good plan?
Simple , logical, flexible, practical, stable and it must be complete and integrated
4. What are Rules ?
Statement of expected results expressed in quantitative terms.
5. Define Procedure
It is a series of related task that up the chronological sequence and the established way of
performing the work to be accomplished.
6. Define Budget.
Statement of expected results expressed in quantitative terms.
7. Define Objectives. (Dec 2009)
Objectives are goals established to guide the efforts of the company and each of its
components
8. What is MBO?
Process whereby the superior and subordinate of an organization jointly identify its common
goal, define each individual’s major areas of responsibility in terms of results expected of
him and use this measures as guides for operating the unit and assessing the contribution of
each if its members
9. What is TOWS matrix ?
It is a conceptual frame work for systematic analysis, which facilitates matching the
external threats and opportunities with the internal weakness and strength of the
organization.
10. Define Strategy. (Dec 2010)
Determination of basic long term objectives and of courses of action and allocation of
resources to achieve these aims.
11. What are Policies ? ( May 2007)
Verbal, written, or implied overall guide setting up boundaries that supply the general limits
and direction in which managerial action will take place.
12. State different strategies of Porter’s generic strategy.
Overall cost leadership strategy, differentiation strategy and focused strategy
13. Define Planning Premises. (May 2008)
Anticipated environment in which plan are expected to operate. They include
assumptions or forecasts of the future and known conditions that will effect the operation
of plans
14. Define MIS. (Dec 2009)
MG2351 Principles of Management

Department of ECE

2013 - 2014

MIS is more advance technology for solving its basic requirements. MIS used for decision
making in the various functional areas of business.
15. Explain the terms decision-making.
It is a process of selection from a set of alternative courses f action which is thought
to
fulfill the objectives of the decision problem more satisfactory than others.
16. What are classification of budget?
a) Functional classification- Sales, production, cash, capital and master budget
b) Time classification- Short
c) current and long term budget
d) Activity level- Fixed and flexible budget
17. What is zero base budget?
Initially the budget is designed from a zero base. The main element is ZBB is future objective
orientation.
18. What is DSS ? (Dec 2010)
DSS –use computers to facilitate the decision making process of semi structured tasks
19. Benefits of planning. (May 2008)
Emphasis on objectives, minimizes uncertainty, facilitates control, improves coordination,
secure economy, encourage innovation and improve competitive strength.
20. What are the purpose of planning?
• To determine the direction of an organization
• To minimize wastages
• To reduce the risk or uncertainty
• To facilitate control.
Part B
1. Explain the Steps in Planning. (Nov 2012)
The basic planning process consists of the following steps: [1] Perception of opportunities, [2]
Establishing objectives, [3] Making planning premises, [4] Identification of alternatives, [5]
Evaluation of alternatives, [6] Choice of appropriate plan, [7] Formulation of supporting plans,
and [8] Establishing action plan for implementation.
2. Explain the types of plans. (May 2013)
I.

II.

Standing plans
• Mission and purpose
• Objectives
• Strategies
• Policies
• Procedures
• Rules
Single use plans
• Programmes
• Budgets
• Schedules
• Methods
• Projects
MG2351 Principles of Management

Department of ECE

2013 - 2014

3. Explain the Tow’s matrix with example.
The letters S, W, O and T respectively stand for Strengths, Weaknesses, Opportunities and Threats. It is a conceptual
framework for a systematic analysis that facilitates matching of the external opportunities and threats with the internal
strengths and weaknesses of the organization. Combining of these factors may require distinct strategic choices.

Internal strengths (S)
Internal weaknesses (W)
External opportunities
SO strategy: Utilize the
WO WO strategy:
(O)
company’s
Development stra strategy to
strengths to take advantage of overcome weakness
opportunities.
to exploit opportunities.
External threats (T)
ST strategy: Use strengths to
WT strategy:
avoid
Retrenchment, liquidation,
or cope with threats.
or joint venture to
minimize weaknesses,
threats.
Recently, the TOWS Matrix concept has been introduced for planning mergers, acquisitions, joint
ventures and alliances. Whenever two partners consider joint activities, it is prudent to analyze the
strengths and weaknesses of each partner as well as their opportunities and threats.
4. Explain the various techniques of decision making.
Explain the salient features of Expected Monetary Value (EMV) criterion and Expected
Opportunity Loss (EOL) criterion.
Explain the salient features of [i] Maximax criterion, [ii] Maximin criterion, [iii] Minimax regret
criterion, [iv] Laplace criterion, and [v] Hurwicz criterion
5. Explain the steps involved in strategy formulation and implementation process. (Nov 2012)
Prof. Michael Porter has suggested that strategy formulation requires an analysis of the attractiveness
of an industry and the company’s position within that industry. This analysis becomes the basis for
formulating generic strategies.
Industry Analysis
In the analysis of the industry, Porter identified five forces: [1] the competition among companies,
[2] the threat of new companies entering the market, [3] the possibility of using substitute products
or services, [4] the bargaining power of the suppliers, and [5] the bargaining power of buyers or
customers. On the basis of such industry analysis, a company may adopt generic strategies. These
strategies are generic because they may be suitable on a broad level for different kinds of
organizations. Any enterprise, however, may use more than one strategy.
1. Overall Cost Leadership Strategy: This strategic approach aims at reduction in costs, based to a
great extent on experience. The emphasis may be on keeping a close watch on costs in areas such as
R&D, operations, sales, and service. The objective is for a company to have a low-cost structure
compared with its competitors. This strategy requires a large market share and cost- efficient
operations. Example: Maruti 800 cars.
2. Differentiation Strategy: A company following this strategy attempts to offer something unique
in the industry in terms of products or services. Example: Dell Computers.
3. Focused Strategy: A company adopting a focused strategy concentrates on special groups of
customers, a particular product line, a specific geographic region, or other aspects that become the
focal point of the firm’s efforts. Rather than serving the entire market with its products or services,
the enterprise may concentrate on a specific segment of the market. A low-cost strategy,
MG2351 Principles of Management

Department of ECE

2013 - 2014

differentiation, or both may accomplish this. An example of focused low-cost strategy is
Businessworld , a competitively priced business magazine. The focused differentiation strategy is
exemplified by Hidesign leather products which specializes in exclusive leather items and manages
to charge premium prices for them.
In general, a company must to choose a generic strategy and should not “get stuck in the middle”,
according to Porter.
6. Write a note on BCG Portfolio matrix.
The Boston Consulting Group (abbreviated BCG) developed the business portfolio matrix. It shows
the linkages between the growth rate of the business and the relative competitive position of the firm,
identified by its market share.
Business Hig
growth
h
Stars
Question
rate
marks
Low

Cash
cows

Dogs

Strong
Weak
Competitive position
Businesses in the ‘question marks’ quadrant, with a weak market share and a high growth rate,
usually require cash investment so that they can become ‘stars’. ‘Stars’ are in the high-growth,
strongly competitive position. These kinds of businesses have opportunities for growth and profit.
The ‘cash cows’, with a strong competitive position and a low growth rate, are usually well
established in the market, and such businesses are in a position to provide cash for the growth of
‘stars’ and ‘question marks’. The ‘dogs’ are businesses with a low growth rate and a weak market
share. These businesses are usually not profitable and should generally be disposed off.
The BCG Matrix was developed for large corporations with several divisions that are often organized
as Strategic Business Units (SBUs).
7. What are the types of decisions?
•
•

Programmed and Non-Programmed Decisions
Major and Minor Decisions
Degree of futurity of decision
Impact of the decision on other functional areas
Recurrence of decisions

•
•

Routine and Strategic Decisions
Individual and Group Decisions

8. What are the types of planning?
• Corporate planning and functional planning
• Strategic planning and operational planning
• Long – term and short – term planning
• Proactive planning and reactive planning
MG2351 Principles of Management
•

Department of ECE

2013 - 2014

Formal and informal planning

9. List out the features of planning.
•
•
•
•
•
•
•
•
•
•

Planning – a primary function
Planning – a dynamic process
Planning – based on objectives and policies
Planning – a selective process
Pervasiveness of planning
Planning – an intellectual process
Planning is directed towards effiency
Planning – focus with future activities
Flexibility of planning
Planning is based on facts.

10. What are the importances of planning?
• Primary of planning
• To offset uncertainty and change
• To focus attention on objectives
• To help in coordination
• To help in control
• To increase organizational effectiveness.
11. What are the steps involved in MBO process? (May 2013)
• Establish long-range objectives and plans consistent with the mission and philosophy of the
organization.
• Establish specific short-term organizational objectives covering each area such as marketing,
profitability, productivity, and so on. These must be supportive of the long range objectives
and plans.
• Establish action plans covering individual performance objectives and standards. It is at this
level that managers and subordinates work closely in setting their individual goals.
Subordinates are given sufficient latitude to devise and implement strategies to achieve these
objectives.
• Periodically appraise results based on a jointly agreed performance appraisal methodology.
• Take appropriate corrective actions if performance deviates from the objectives set.
UNIT III
Part A
1. Define Organizing?
Organizing is the process of identifying and activities required to attain the objectives, delegating , creating
responsibility and establishing relationships I people to work effectively.
2. Mention any four characteristics of an organization.
a. Common objectives
b. Specialisation or Division of Labour
c. Authority of structure
d. Group of persons
MG2351 Principles of Management

Department of ECE

2013 - 2014

3. List out the steps involved in organization process.
a. Determination of activities
b. Grouping of activities
c. Assignment of Duties
d. Delegation of authority
4. Mention the various principles involved in organization.
1. Principle of unity of objective
2. Principle of division of work or specialization
3. Principle of efficiency
4. Principle of span of control
5. Differentiate between formal and informal organization.
Point of
SI no.
Formal organisation
view

Informal organisation

It is created deliberately and
consciously by the frames of the
organisation.
It is created for achieving legitimate
objectives of the organisation.

It is created spontaneously
naturally.

Nature

Planned and official

Unplanned and unofficial.

Size

It may quite large.

It may be small size.

1.

Origin

2.

Purpose

3.
4.

It is created by members of the
organisation for social and
psychological satisfaction.

6. Mention the three categories of relationships in span of management.
a. Direct single relationship
b. Direct group relationships
c. Cross relationship
7. State the important factors in determining an effective span of management.
1. Capacity of superior
2. Capacity of subordinates
3. Nature of work
4. Type of technology
5. Delegation of authority
8. What are the types of departmentation ? (May 2008)
1. Departmentation by function.
2. Departmentation by Territory or Geography.
3. Departmentation by Customers.
4. Departmentation by equipment or process.
5. Departmentation by product or service.
9. State the advantages of departmentation by function. (Dec 2009)
Advantages:
1. It is most logical, scientific, time proven and natural method of departmentation.
2. It provides specialization of work which makes maximum utilization of manpower
and other resources.
3. It ensures proper performance control.
MG2351 Principles of Management

Department of ECE

2013 - 2014

4. It facilitates delegation of authority and therefore, reduces the burden of top executives.
10. State the disadvantages of departmentation by function. (May 2010)
Disadvantages:
1. There is a tendency for overspecialization. The department managers
are experts in handling the problems in their department alone. They may not be
able to understand the problem of other departments.
2. Functional departmentation discourages communication across functions so that the
workers develop a narrow technical point of horizontal conflicts.
3. It increases the workload and responsibility of the departmental heads.
4. It does not offer any scope for training for the overall development of manager
11. Give a note on departmentation by customers.
This type of departmentation is preferred when the needs of customers are different in nature. Big
organizations provide special services to different types of customer.
12. What is departmentation by product?
Departmentation By Product Or Service: In this case, the units are formed according to the product.
It is more useful in multi-line corporations where ] expansion and diversification, manufacturing
and mark characteristics of the product are of primary concern.
13. Define authority. (May 2009)
"Authority is the right to give orders and the power to exact obedience".
14. List out the sources of authority.
a. Formal authority theory
b. Acceptance Authority Theory
c. Competence theory
15. What is line authority?
Line authority is the direct authority which a superior exercises over a number of subordinates
to carry out orders and instructions. In an organizing process, authority is delegated to the individuals to
perform the activities.
16. What is staff authority?
A staff person assists the line people in attaining their objectives. Staff authority is purely advisory. Types
of staff authority are a) Advisory staff authority, b) Compulsory staff authority & c) Concurrent staff authority.
17. State the types of organizational charts.
a) Vertical or Top-to-Bottom chart
b) Horizontal or Left-to-right chart
c) Circular or concentric chart
18. What is departmentation by process?
Departmentation By Process Or Equipment: Under this type of departmentation, activities are grouped
on the basis of production processes or equipment involved.
19. What are various stages of staffing Process?
MG2351 Principles of Management

Department of ECE

2013 - 2014

External environment
Enterprise
plans

Organization
plans

Number and kinds
of managers
required

External
sources
Recruitment
Selection
Placement
Promotion
Separation

Analysis of
present and
future needs
for managers
Internal
sources

Appraisal
Career
Strategy
Training
&Develop
ment

Manager
inventory

20. What is Performance Appraisal? What are the different methods/techniques of
Performance appraisal? (May 2012)
Performance Appraisal is evaluating the actual performance of the employee for determining
the compensation and identifying the potential of the
employees. Methods/Techniques
of Performance Appraisal: (i) Trait-based appraisal:(job knowledge, leadership, judgment,
ability, initiative,
loyalty): Graphic scale method, ranking method, grading system,
forced distribution method, check list method, Critical incident method, group appraisal
(ii)Appraisal by results: MBO, Behaviorally Anchored Rating(BARS), Assessment Centre,
360 degree performance appraisal.
Part B
1. Differentiate formal and informal organization.
Formal organization

Informal organization

1. It is a prescribed structure of roles and
1. It is a natural and spontaneous structure,
relationships consciously created to arising out of the social tendency of people to
achieve
associate and interact. Management has no
a common objective.
role in its emergence and functioning.
2. Its values, goals, tasks are oriented Its values, goals, tasks centre around
towards
individual and group satisfaction, esteem,
productivity, profitability, efficiency, etc.
affiliation, friendship, etc.
3. It is well-defined in shape. Majority of
formal organizations are pyramid-shaped.
Ranks of individuals are made clear by the
use of titles.
4. There is a prescribed, mostly written
system of reward and punishment.
Rewards can be both monetary and non-

It is shapeless. There are a number of multidirectional, intricate relationships which
cannot be easily charted.
There is an unwritten system of reward and
punishment. Rewards take the form of
continued membership, social esteem,
MG2351 Principles of Management

Department of ECE

monetary.

2013 - 2014

satisfaction,
group
leadership,
etc.
Punishments
are
isolation,
censure,
harassment, etc.
5. This organization is usually very This organization is not very enduring, being
enduring and may grow to any size.
dependent upon the sentiments of members,
which often change. It also tends to remain
small.
2. Discuss art of delegation of authority. (Nov 2012)
Personal attitude towards delegation
•

Receptiveness

•

Willingness to let go

•

Willingness to let other make mistake

•

Willingness to trust subordinates

•

Willingness to establish and use broad controls

Guidelines for overcoming weak delegation
•

Defined assignments

•

Selection of appropriate person

•

Open lines of communication

•

Proper control

•

Rewarding

3. List out the steps involved in selection process.
• Issuing blank application
•

Initial interview

•

Employment test

•

Checking references

•

Physical and medical examination

•

Final interview

4. Explain the training methods in detail. (May 2012)
On the job training method
•

Job rotation

•

Apprenticeship and coaching

•

Committee assignments

•

Experience
MG2351 Principles of Management
•

Department of ECE

2013 - 2014

Temporary promotions

Off – the job training method
•

Lecture

•

Conference and seminars

•

Role playing

•

Case studies

•

Programmed instructions

•

Business games

•

In basket method

•

Sensitivity training

5. What are the types of organizational structure?
1. Line organization: It is also known as scalar, military, or vertical organization. It
is the oldest form of organization structure. It is most common among small
companies. The authority flows in a direct line from the top management down to
different levels of managers and subordinates down to the operative level of workers.
Merits: 1. Simplicity, 2. Discipline, 3. Prompt decisions, 4. Orderly communication,
5. Easy supervision and control.
Demerits: 1. Autocratic approach, 2. Problems of coordination, 3. Not suited for
large, complex organizations.
2. Line and staff organization: A pattern in which staff specialists assist and advise
line managers to perform their duties. The staff positions or departments are generally
of advisory nature. The staff specialists may provide services to a particular position,
department, or the organization as a whole.
Merits: 1. Planned specialization, 2. Quality decisions, 3. Prospect for personal
growth, 4. Training ground for personnel.
Demerits: 1. Lack of well defined authority, 2. Potential for line-staff conflicts.
3. Functional organization: It is the most widely used structure in medium and large
organizations making limited number of products. It is created by grouping activities
on the basis of functions such as production, marketing, finance, etc. Several
departments may be created by dividing the basic functions further into sub-functions,
e.g. marketing into sales, distribution, advertising, etc. Staff positions may be created
to assist the line functions.
Merits: 1. Expertise through specialization, 2. Order and clarity, 3. Good
coordination and control, 4. Economy in using resources, 5. Promotes professional
achievement.
Demerits: 1. Focus on departmental goals, 2. Potential for conflicts between
departments.
4. Divisional organization: Growth of an organization through geographic and
product diversification requires the adoption of the divisional structure. In this form,
the organization is divided into several autonomous business units. Each unit is selfcontained and independent. Each division has its own manufacturing, marketing, etc.
departments. The various divisions may be created on the basis of products,
territories, or customers. For example, Hindustan Unilever has personal care, home
care, food products, etc. divisions, each offering a related range of products. Indian
Railways is organized on the basis of Northern Railway, Southern Railway, etc. based
on regions. Airtel has separate divisions to cater to corporate customers and retail
customers.
MG2351 Principles of Management

Department of ECE

2013 - 2014

Merits: 1. Efficient way to manage diversified activities, 2. Better focus on results, 3.
Greater flexibility, 4. Better customer service.
Demerits: 1. Duplication of facilities, 2. Complex control systems.
5. Project organization: It is a form of divisional organization suitable for
companies which have to execute major projects. Each project is managed as a
separate division which exists during the life time of the project. When a particular
project is completed, the concerned division may cease to exist. Each project is
headed by a project manager who coordinates the activities of the project and is
responsible for the completion of the project. The staff for each project are drawn
from the from various functional departments and report to the project manager. Once
the project is completed, they go back to their respective departments. Examples of
projects are civil construction contracts, new product development, etc.
Merits: 1. Maximum use of specialized knowledge and skills, 2. Flexibility in using
resources.
Demerits: Problems of dual authority and responsibility.
6. Matrix organization: It is somewhat similar to project organization. The basic
difference is that project organization is suitable for taking up a small number of
large, long duration projects; matrix organization is suitable for taking up a large
number of small, short duration projects. The activities of various such projects can
be accomplished through temporary departments. Matrix organization is twodimensional in structure; the project structure and functional structure are merged
together to create the matrix structure. Here also, a project manager is appointed to
coordinate the activities of the project. Personnel are drawn from their respective
functional departments. Upon completion of the project, these people may return to
their original departments for further assignment. Thus each functional staff has two
bosses – his administrative head and his project manager.
Merits: 1. Maximum use of specialized knowledge and skills, 2. Flexibility in using
resources.
Demerits: Problems of dual authority and responsibility.
6. What are the types of departmentation? (May 2013)
Departmentation based on functions
Each major function of the enterprise is grouped into a department. For example, there may be
production, finance and marketing departments in a manufacturing company, or underwriting and
claims departments in an insurance company. A sales manager in this kind of department ation is
responsible for the sale of all products manufactured by the company or a claims manager is
responsible for dealing with all claims from different areas where the company operates. The relevant
organization structure is functional organization.
Advantages:
1. It is a simple form of grouping activities for organizations which manufacture only a limited number
of products or render only a limited number of services.
2. It promotes excellence in performance because of development of expertise through
specialization..
3. It leads to improved planning and control of the key functions.
4. It ensures economy, for there is only one department related to one function for the entire
company. Manpower and other resources of the company are effectively utilized by sharing them
across products or projects.
Drawbacks:
1 . I t fosters sub-goal loyalties. Each manager thinks only in terms of his own departmental goals
and does not think in terms of the company as a whole. This results in inter-departmental conflicts and
disagreements
2. It does not offer a good training ground for the overall development of a manager. The
manager gains expertise in handling problems of his particular department only.
3. It is unsuitable where either geographical dispersal of units is required or emphasis on
separate product lines is called for.
MG2351 Principles of Management

Department of ECE

2013 - 2014

2. Departmentation hased on products
This form is suited for large organizations manufacturing a variety of products. Under this
method, for each major product, a separate department is created and is put under the charge
of a manager. He may also be made responsible for producing a given level of profit. Within
each department, all the needed manufacturing, engineering, marketing, manpower and other
facilities are created. Product departmentation is the logical pattern to follow when each product
requires raw material, manufacturing technology, and marketing methods that are markedly
different from those used by other products in the organisation. Examples are: Hindustan
Unilever, Godrej, ITC, Reliance Industries, etc. The relevant organization structure is
divisional organization.
Advantages
1. This form relieves top management of operating task responsibility. It can therefore better
concentrate on such centralised activities like finance, R & D and control.
2. This form enables top management to compare the performances of different products
and invest more resources in profitable products and withdraw resources from unprofitable
ones.
3. In this form, since the responsibility for each product's performance is entrusted to a
particular departmental head, he is better stimulated to improve his performance.
4. In this form, those who work within a department derive greater satisfaction from
identification with a recognisable goal.
Drawbacks
1. This form results in duplication of staff and facilities.
2. Extra expenditure is incurred in maintaining a sales force for each product line.
3. Employment of a large number of managerial personnel is required.
4. Equipment in each product department may not be used fully.
3. Departmentation based on customers
An enterprise may be divided into a number of departments on the basis of the customers that it
services. For example, an electronics firm may have separate departments for military, industrial and
consumer customers. A big automobile servicing enterprise may have separate departments for
servicing cars, heavy vehicles and scooters, or an educational institution may have separate
departments for day, evening and correspondence courses to impart education to full-time students,
locally employed students and outstation students, respectively.
One big advantage of this form is that it ensures full attention to major customer groups and this
helps the company to earn goodwill. The drawbacks of this form are that (i) it may result in underutilisation of resources and facilities in some departments; and (ii) there may be duplication of
facilities. Example is Airtel. The relevant organization structure is divisional organization.
4. Departmentation based on Regions or Territories
When several production or marketing units of an organisation are geographically dispersed in
various locations, it is logical to departmentalise those units on a geographical basis. The Indian
Railways are departmentalised on this basis. Northern Railways, Western Railways, Southern
Railways, Eastern Railways, Central Railways, etc. are departments in this sense. Here also the
relevant organization structure is divisional organization.
Advantages
1. It motivates each regional head to achieve high performance.
2. It provides each regional head an opportunity to adapt to his local situation and customer need
with speed and accuracy.
3. It affords valuable top-management training and experience to middle-level executives.
4. It enables the organization to take advantage of locational factors, such as availability of raw
materials, labour, market, etc.
5. It enables the organization to compare regional performances and invest more resources in
profitable regions and withdraw resources from unprofitable ones.
MG2351 Principles of Management

Department of ECE

2013 - 2014

Drawbacks
1. It gives rise to duplication of various activities. Many routine and service functions performed
by all the regional units can be performed centrally by the head office very economically..
2. Various regional units may become so engrossed in short-run competition among themselves
that they may forget the overall interest of the total organization.
6. Departmentation based on process
Departmentation is here done on the basis of several discrete processes or technologies involved
in the manufacture of a product. For example, a concern engaged in the production of vegetable
oil may have separate departments for crushing, refining and finishing. Similarly a cotton textile
mill may have separate departments for ginning, spinning, weaving, dyeing, printing, packing
and sales. In this way, whenever work that would otherwise be done in several different locations
in an enterprise is done in one place. Here also the relevant organization structure is divisional
organization.
Advantages
1. It facilitates the use of heavy and costly equipment in an efficient manner. There is very little
chance for the equipment remaining idle or under-utilized because there is no duplication of the
same.
2. It follows the principle of specialization— each department is engaged in doing a special type
of work. This increases efficiency.
3. It is suitable for organizations which are engaged in the manufacture of products which
involve a number of processes.
Drawbacks
1. It does not provide good training ground and opportunity for the overall development of
managerial talent.
2. When the process is sequential, the dependent departments generally become hostile to
other departments and they express their resentment either by complaining directly about
other departments or by passing on inferior work to their successive departments.
3. It is difficult to compare the performance of different process-based departments.
7. Combined Base Departmentation
It is quite typical to find an organization following a different base of departmentation at different
organisational levels. For example, an organization manufacturing agricultural machinery may
follow 'product' as the base (tractor department, appliance department, generator department, etc)
at the primary level (i.e., the level immediately below the chief executive), 'territory' as the base
at the intermediate level and 'function' as the base at the ultimate level.
Another form of combined base organization which is becoming very popular nowadays is matrix
organization. In this form of organization (also called grid or lattice pattern), two types of
departmentation—say, functional and product—may exist simultaneously.
7. Explain different methods of appraisal system. (Nov 2011)
(i) Traditional methods
• Graphic rating scales
• Rankling method
• Paired comparison method
• Forced distribution method
• Checklist methods
• Essay form appraisal
• Group appraisal
MG2351 Principles of Management
•

Department of ECE

2013 - 2014

Confidential reports

(ii) Modern methods
• Behaviorally anchored rating scales
• Assessment centres
• Human – asset – accounting method
• Management by objectives
• 360º performance appraisal
8. What are the factors affecting organization structure?
• Environment ( Mechanistic and organic system)
• Strategy
• Technology
• People
• Size of the firm.
9. What are the sources of recruitment?
1. Internal sources
• Transfer and promotions
• Job posting
• Employee referrals
II. External sources
• Direct methods
• Indirect methods
• Third party methods
10. What are the factors affecting span of management?
• Capacity of supervisor
• Capacity of subordinate
• Nature of work
• Degree of decentralization
• Degree of planning
• Communication techniques
• Use of staff assistance
• Supervision from others.
UNIT – IV
Part A
1. Define Leading.
It is a process of influencing people so that they will contribute to organization and group goals.
2. Define Motivation. (May 2008)
It is a process of stimulating people to action to achieve/ accomplish desired goals. Motivation is
defined as “those forces operating within the individual employee or subordinate which impel
him to act or not to act in certain ways:. Motivation is mainly psychological.
3. Democratic leader. (Dec 2010)
MG2351 Principles of Management

Department of ECE

2013 - 2014

He consults with subordinates on proposed actions and decisions and encourages participation
from them.
.4. Define Job Enrichment.
It attempts to make a job more varied by removing the dullness associated with performing
repetitive operations
5. What is QWL?
Increasing productivity and reducing inflation and as a way of obtaining industrial democracy
and minimizing labor disputes.
6. Define Leadership.
Art or process of influencing people so that they will strive willingly and enthusiastically
towards achievement of the goal
.
7. What are the Styles of Leadership ?
• Autocratic leadership
• Democratic leadership and
• Free-rien leadership
8. Define Communication.
Communication is the process of transmitting ideas, facts, opinions and feelings to others.
Communication is a mutual interchange process that occurs between two or more persons.
9. What are the various types of communication? (Dec 2009)
Formal,informal,vertical,horizontal,grapevine,written and oral communication.
10. What is Country club management?
Mangers have little or no concern for production. but are concerned for people
11. What are the types of needs?
• Primary Needs: Physiological requirements such as food, sleep, water and shelter
•

Secondary needs: Self esteem, status, affection, accomplishment and affiliation with others.

12. Explain - Maslow’s Needs. (May 2010)
Physiological needs, Safety needs, Social needs, Self- Esteem needs and Self- ctualization needs.
13. Define Incentives
An incentive is something which stimulates a person towards some determination
14. What are Job content factors?
• Achievement, advancement, responsibility,
• Respect, growth and development.
15. Define Expectancy theory.
Force=Valance * Expectancy
16. What is an Equity theory?
Outcomes by a person
Outcomes by another person
------------------------------------ =
----------------------------------Inputs by a person
inputs by another person
17. Define Creativity.
Ability and power to develop new ideas
18. What is Innovation.
Use of new ideas.
19. Define Brainstorming
Improve problem solving by finding new ideas and unusual solution
20. What are the importance of leadership?
Securing Co-operation, Creating confidence in the minds of employees, Providing good
working climate.
MG2351 Principles of Management

Department of ECE

2013 - 2014

Part B
1. Write short notes on the managerial grid.
A well-known approach to defining leadership styles is the managerial grid, developed some years
ago by Robert Blake and Jane Mouton. Building on previous research that showed the importance of
a manager's having concern both for production and for people, Blake and Mouton devised a clever
device to dramatize this concern. This grid, shown in the figure, has been used throughout the world
as a means of training managers and of identifying various combinations of leadership styles.
Grid Dimensions: The grid has two dimensions: concern for people and concern for production.
As Blake and Mouton have emphasized, the phrase "concern for" is meant to convey "how"
managers are concerned about production or "how" they are concerned about people, and not such
things as "how much" production they are concerned about getting out of a group.
"Concern for production" includes the attitudes of a supervisor toward a wide variety of things,
such as the quality of policy decisions, procedures and processes, creativeness of research, quality of
staff services, work efficiency, and volume of output. "Concern for people" is likewise interpreted in
a broad way. It includes such elements as degree of personal commitment toward goal achievement,
maintenance of the self-esteem of workers, placement of responsibility on the basis of trust rather
than obedience, provision of good working conditions, and maintenance of satisfying interpersonal
relations.
Four Extreme Styles: Blake and Mouton recognize four extremes of style. Under the 1.1 style
(referred to as "impoverished management"), managers concern themselves very little with either
people or
production and have minimum involvement in their jobs; to all intents and purposes, they have
abandoned their jobs and only mark time or act as messengers communicating information from
superiors to subordinates. At the other extreme are the 9.9 managers, who display in their actions the
highest possible dedication both to people and to production. They are real "team managers" who
are able to mesh the production needs of the enterprise with the needs of individuals.
Another style is 1.9 management (called "country club management" by some), in which
managers have little or no concern for production but are concerned only for people. They promote
an environment in which everyone is relaxed, friendly, and happy and no one is concerned about
putting forth coordinated effort to accomplish enterprise goals. At another extreme are the 9.1
managers (sometimes referred to as "autocratic task managers") who are concerned only with
developing an efficient operation, who have little or no concern for people, and who are quite
autocratic in their style of leadership.
By using these four extremes as points of reference, every managerial technique, approach, or style
can be placed somewhere on the grid. Clearly, 5.5 managers have medium concern for production
and for people. They do not set goals too high, and they are likely to have a rather benevolently
autocratic attitude toward people.
The managerial grid is a useful device of identifying and classifying managerial styles, but it does
not tell us why a manager falls into one part or another of the grid. To determine the reason, one has
to look at underlying causes, such as the personality characteristics of the leader or the followers, the
ability and training of managers, the enterprise environment, and other situational factors that
influence how leaders and followers act.
2. What are the steps in communication process? (May 2013)
Communication has been defined as a process. The term process refers to identifiable flow of
information through interrelated stages of analysis directed towards the achievement of an objective.
The various elements of the communication process are sender, message, encoding, channel,
receiver, decoding, understanding and feedback.
MG2351 Principles of Management

Department of ECE

2013 - 2014

Sender. Sender of the message is the person who intends to make contact with the objective of
passing the message to other persons. In organisational context, sender may be a superior, a
subordinate, a peer, or any other person. The organisational position of the sender determines the
direction of flow of communication in an organisation.
Message. This is the subject-matter of the communication which is intended to be passed to the
receiver from the sender. The message may be in the form of thoughts, ideas, opinions, feelings,
views, suggestions, orders, etc.
Encoding. Since the subject-matter of communication, viz. thought, idea, etc., is abstract and
intangible, its transmission requires the use of certain symbols such as words, pictures, gestures, etc.
The process of converting the message into communication symbols is known as encoding.
Channel. Message encoded into symbols is transmitted by the sender through a channel like written
form, personal contact, phone call, e-mail, etc. depending on the two parties—the sender and
receiver.
Receiver. Receiver is the person to whom the symbols are transmitted. Like sender, he may be a
superior, subordinate, peer, or any other person in the organisation.
Decoding. Decoding is the reverse of encoding. The receiver receives the subject matter of
communication in the form of communication symbols in which the sender has encoded his message.
The receiver decodes these symbols into message.
Understanding. Interpreting, obtaining meaning from the message communicated.
Feedback. Feedback is necessary to ensure that the receiver has received the message and
understood it in the same sense as the sender intended. The sender can never be sure whether or not
his message has been effectively encoded, transmitted, received, decoded, and understood until it is
confirmed by feedback.
3. Explain the motivation theories. (Nov 2012)
1. MASLOW’S HIERARCHY OF NEEDS THEORY
One of the most widely mentioned theories of motivation is the hierarchy of needs theory put forth
by psychologist Abraham Maslow. Maslow saw human needs in the form of a hierarchy, ascending
from the lowest to the highest, and he concluded that when one set of needs is satisfied, the next
higher level need gets activated.
The basic human needs placed by Maslow in an ascending order of importance are:
1. Physiological Needs These are the basic needs for sustaining human life itself, such as food,
water, warmth, shelter, and sleep. Maslow felt that until these needs are satisfied to the degree
necessary to maintain life, other needs will not motivate people.
2. Security or Safety Needs These are the needs to be free of physical danger and of the fear of
losing a job property, food, or shelter.
3. Affiliation or Social Needs Since people are social beings, they need to belong, to be accepted by
others. It includes friendship, the need to love and be loved, socializing, etc.
4. Esteem Needs Once people begin to satisfy their need to belong, they tend to want to be held in
esteem both by themselves and by others. This kind of need produces such satisfactions as respect,
power, prestige, status, and self-confidence.
5. Self-actualizationNeeds This as the highest need in the hierarchy. It is the desire to become what
one is capable of becoming—to fully realize one's potential and to accomplish what one is capable of
achieving.
Maslow suggests that the various levels are overlapping, each higher-level need emerging before the
lower-level need has been completely satisfied. Since one need does not disappear when another
MG2351 Principles of Management

Department of ECE

2013 - 2014

emerges, all needs tend to be partially satisfied in each area. When the peak of a need is passed, that
need ceases to be the primary motivator. The next level need then begins to dominate.
2. HERZBERG’S MOTIVATION-HYGIENE THEORY
Maslow's need approach has been considerably modified by Frederick Herzberg. His research
purports to find a two-factor theory of motivation. In one group of needs are such things as company
policy and administration, supervision, working conditions, interpersonal relations, salary, status, and
job security. These were found by Herzberg and his associates to be only dissatisfiers and not
motivators. Their existence does not motivate in the sense of yielding satisfaction; their lack of
existence would, however, result in dissatisfaction. Herzberg called them maintenance, hygiene or
job context factors.
In the second group, Herzberg listed certain satisfiers—and therefore motivators—all related to job
content. They include achievement, recognition, challenging work, advancement, and growth in the
job. Their existence will yield feelings of satisfaction but their absence will not lead to
dissatisfaction. The satisfiers and dissatisfiers identified by Herzberg are similar to the factors
suggested by Maslow: the dissatisfiers correspond to Maslow’s lower order needs (physiological,
safety and social needs) while the satisfiers correspond to Maslow’s higher order needs (esteem and
self-actualization needs).
The first group of factors (the dissatisfiers) will not motivate in an organization; yet they must be
present, or dissatisfaction will arise. The second group, or the job content factors, Herzberg found to
be the real motivators because they have the potential of yielding a sense of satisfaction.
3. VROOM’S EXPECTANCY THEORY
Another approach to motivation is the expectancy theory of the psychologist Victor H. Vroom. He
holds that people will be motivated to do things to reach a goal if they believe in the worth of that
goal and if they can see that what they do will help them in achieving it. Vroom's theory is that
people's motivation toward doing anything will be determined by the value they place on the
outcome of their effort (whether positive or negative), multiplied by the confidence they have that
their efforts will materially aid in achieving a goal. In other words, Vroom makes the point that
motivation is a product of the anticipated worth that an individual places on a goal and the chances
he or she sees of achieving that goal. Using his own terms, Vroom's theory may be stated as
Force = valence x expectancy
Where force is the strength of a person motivation, valence is the strength of an individual's
preference for an outcome, and expectancy is the probability that a particular action will lead to a
desired outcome.
When a person is indifferent about achieving a certain goal, a valence of zero occurs; there is a
negative valence when the person would rather not achieve the goal. The result of either would be, of
course, no motivation. Likewise, a person would have no motivation to achieve a goal if the
expectancy were zero or negative. Hence the force exerted to do something will depend on both
valence and expectancy. Moreover, a motive to accomplish some action might be determined by a
desire to accomplish something else. Vroom identified these as first-level and second-level
outcomes. For example, a person might be willing to work hard to please his boss (first-level
outcome) in order to get higher pay (second-level outcome). Or a manager might be willing to work
hard to achieve company goals (first-level outcome) for the sake of getting a promotion (secondlevel outcome). Vroom used the term ‘instrumentality’ to indicate the extent to which the
achievement of first-level outcomes lead to the achievement of second-level outcomes.
4. PORTER AND LAWLER MODEL
MG2351 Principles of Management

Department of ECE

2013 - 2014

Lyman W. Porter and Edward E. Lawler III derived a substantially more complete model of
motivation, built in large part on Vroom’s expectancy theory. In their study, they have applied this
model primarily to managers.
As the model indicates, the amount of effort (the strength of motivation and energy exerted)
depends on the value of a reward plus the amount of energy a person believes is required and the
probability of receiving the reward. The perceived effort and probability of actually getting a reward
are, in turn, also influenced by the record of actual performance. Clearly, if people know they can do
a job or if they haw done it before, they have a better appreciation of the effort required and know
better the probability of rewards.
Actual performance in a job (the doing of tasks or the meeting of goals) is determined principally
by effort expended. But it is also greatly influenced by an individual's ability (knowledge and skills)
to do the job and by his or her perception of what the required task is (the extent to which the person
understands the goals, required activities, and other elements of a task). Performance, in turn, is seen
as leading to intrinsic rewards (such as a sense accomplishment or self-actualization) and extrinsic
rewards (such as working conditions and status). These rewards, tempered by what the individual
sees as equitable, lead to satisfaction. But performance also influences sensed equitable rewards.
Understanding what the individual sees as a fair reward for effort will necessarily affect the
satisfaction derived. Likewise, the actual value of rewards will be influenced by satisfaction.
5. ADAMS’ EQUITY THEORY
An important factor in motivation is whether individuals perceive the reward structure as being fair.
One way of addressing this issue is through equity theory, which refers to an individual's subjective
judgments about the fairness of the reward she or he got, relative to the inputs (which include many
factors such as effort, experience, education, and so on), in comparison with the rewards of other. J.
Stacy Adams has formulated the equity theory. The essential aspects of the equity theory may be
shown as follows.
Outcomes by a person = Outcomes by another person
Inputs by a person
Inputs by another person
There should be a balance of the outcomes-inputs relationship for one person in comparison with
that for another person.
If people feel they are inequitably rewarded, they may be dissatisfied, reduce the quantity or quality
of output, or leave the organization. They also can ask for a greater reward. If people perceive the
rewards as equitable, they probably will continue at the same level of output. If people think the
rewards are greater than what is considered equitable, they may work harder. It is also possible that
some may discount the reward.
The impact of inequity on a person is as follows:
1. Perceived inequity creates tension in the person.
2. The amount of tension is proportional to the magnitude of the inequity.
3. The tension created in the person will motivate him to reduce it.
4. The strength of the motivation to reduce inequity is proportional to the perceived inequity.
6. SKINNER’S REINFORCEMENT or BEHAVIOR MODIFICATION THEORY
This theory has developed out of researches done by B.F. Skinner. According to this theory, people
behave the way they do because, in past circumstances, they have learned that certain behaviors are
associated with pleasant outcomes and certain other behaviours are associated with unpleasant
outcomes. Because people generally prefer pleasant outcomes, they are likely to repeat behaviour
that they have learned will have pleasant consequences. For example, people in an organisation are
likely to obey a manager's instruction because they have learned at home and at school that
obedience to authority leads to praise and disobedience leads to punishment. Thus, the frequency of
MG2351 Principles of Management

Department of ECE

2013 - 2014

the various kinds of behaviour in the organisation can be seen as contingent on the immediate
consequences of these behaviours.
The consequences that increase the frequency of a behaviour are positive reinforcement (e.g., praise
or monetary reward) or negative reinforcement (i.e., escape from some aversive situation). An
example of negative reinforcement might be a manager who requires all subordinates to attend an
early morning staff meeting whenever performance of the organisation falls below a certain level.
Subordinates would then work very hard to avoid the unpleasant early morning meetings and strive
for a high level of performance.
The consequences that decrease the frequency of a behaviour are extinction (i.e., ignoring the
behaviour) and punishment (e.g. reprimand, fine, frown etc.). When reward or positive reinforcement
is being used to increase the frequency of a desired behaviour the important point to be considered
by the manager is the frequency of reward. Rewards can be administered continuously—every
desired behaviour can be reinforced—or they can be administered on an intermittent basis. Normally
a manager begins by reinforcing every desired behaviour. A secretary may be complemented every
time he types an error-free letter. Then as the behaviour becomes permanent, the rewards are
gradually tapered off to an intermittent or random basis. Instead of praising the secretary for every
error-free letter, the manager may begin skipping now and then.
7. McCLELLAND'S NEEDS THEORY
David C. McClelland has contributed to the understanding of motivation by identifying three types
of basic motivating needs. He classified them as the need for power (n/PWR), need for affiliation
(n/AFF), and
need for achievement (n/ACH).
Need for Power
McClelland and other researchers have found that people with a high need for power have a great
concern for exercising influence and control. Such individuals generally are seeking positions of
leadership; they are frequently good conversationalists, though often argumentative; they are
forceful, outspoken, hard-headed, and demanding; and they enjoy teaching and public speaking.
Need for Affiliation
People with a high need for affiliation usually derive pleasure from being loved and tend to avoid the
pain of being rejected by a social group. As individuals, they are likely to be concerned with
maintaining pleasant social relationships, to enjoy sense of intimacy and understanding, to be ready
to console and help others in trouble, and to enjoy friendly interaction with others.
Need for Achievement
People with a high need for achievement have an intense desire for success and an equally intense
fear of failure. They want to be challenged, and they set moderately difficult (but not impossible)
goals for themselves. They take a realistic approach to risk; they are not likely to be gamblers but,
rather, prefer to analyze and assess problems, assume personal responsibility for getting a job done,
and like specific and prompt feedback on how they are doing. They tend to be restless, like to work
long hours, do not worry unduly about failure if it does occur, and tend to like to run their own
shows.
The implication of this theory is that all three drives—power, affiliation, and achievement—are of
relevance to management, since all must be recognized to make an organized enterprise work well.
Because any organized enterprise and every department in it represents groups of individuals
working together to achieve goals, organizations can improve the selection and placement processes
if the needs of employees can be accurately assessed.
8. ALDERFER'S ERG THEORY
MG2351 Principles of Management

Department of ECE

2013 - 2014

Alderfer has provided an extension of the Maslow's need hierarchy and Herzberg's two-factor theory
of motivation, particularly the former. Like the previous theories, Alderfer believes that there is a
value in categorising needs and that there is a basic distinction between lower-order needs and
higher-order needs. Based on the empirical evidences, he has found that there seems to be some
overlapping between physiological, security and social needs. Also the lines of demarcation between
social, esteem and achievement needs are not clear. Based on these observations, Alderfer has
categorised the various needs into three categories; existence needs, relatedness needs, and growth
needs. The first three letters of these needs are used to call it ERG theory.
Existence Needs. Existence needs include all needs related to physiological and safety aspects of an
individual. Thus, existence needs group physiological and safety needs of Maslow into one category
as these have similar impact on the behavour of the individual.
Relatedness Needs. Relatedness needs include all those needs that involve relationship with other
people whom the individual cares. Relatedness needs cover Maslow's social needs and that part of
esteem needs which is derived from the relationship with other people.
Growth Needs. Growth needs involve the individual making creative efforts to achieve full potential
in the existing environment. These include Maslow's self-actualisation need as well as that part of the
esteem need which is internal to the individual like feeling of being unique, feeling of personal
growth, etc.
ERG theory offers the following propositions so far as satisfaction of various needs is concerned:
1. The three need categories form a hierarchy only in the sense of decreasing concreteness. As people
move from a focus on existence to relatedness to growth needs, the ways in which they can satisfy
those needs become increasingly abstract.
2.
The rise in the level of satisfaction of any lower-order need may result in decrease in its
importance. Its place is taken by another need. Thus, the individual is able to move to become
productive and creative, and as he moves to this level, he sets a higher goal for himself.
3. People are likely to try to satisfy their most concrete needs first and then, they move on to the
abstract needs. In this way, progression of need satisfaction of ERG theory is similar to Maslow's
need hierarchy, that is, people first satisfy their lower needs and gradually progress to the satisfaction
of higher needs, people can experience frustration-regression, that is, if people cannot satisfy their
needs at a given level of abstraction, they 'drop back' and again focus on more concrete needs.
9. McGREGOR'S THEORY X AND THEORY Y
The management's action of motivating human beings in the organisation, according to Douglas
McGregor, involves certain assumptions, generalizations and hypotheses relating to human
behaviour and human nature. They serve the purpose of predicting human behaviour. McGregor has
characterised these assumptions in two opposite views, termed Theory X and Theory Y.
Theory X. This is the traditional theory of human behaviour, In this theory, McGregor has certain
assumptions about human behaviour. These assumptions are as follows:
1. Management is a process of directing employees’ efforts, motivating them, controlling their
actions, modifying their behaviour to fit the needs of the organisation.
2. Without this active intervention by management, people would be passive— even resistant—to
organisational needs. They must be persuaded, rewarded, punished, controlled, and their activities
must be directed.
3. The average man is by nature indolent—he works as little as possible.
4. He lacks ambition, dislikes responsibility, prefers to be led.
5. He is inherently self-centered, indifferent to organisational needs.
6. He is, by nature, resistant to change.
7. He is gullible, not very bright, the ready dupe of the charlatan and the demagogue.
MG2351 Principles of Management

Department of ECE

2013 - 2014

These assumptions about human nature are negative in their approach, however much
organisational processes have developed on these assumptions. Managers subscribing to these views
about human nature attempt to structure, control and closely supervise their employees. They feel
that external control is most appropriate for dealing with irresponsible and immature employees.
Theory Y. The assumptions of Theory Y are described by McGregor in the following words:
1. The expenditure of physical and mental effort in work is as natural as play or rest. The average
human being does not inherently dislike work. Depending upon controllable conditions, work may
be a source of satisfaction or a source of punishment.
2. External control and the threat of punishment are not the only means for bringing about effort
towards organisational objectives. Man will exercise self-direction and self-control in the service of
objectives to which he is committed.
3. Commitment to objectives is a function of the reward associated with their achievement. The most
significant of such awards, e.g. the satisfaction of ego and self-actualisation needs, can be a direct
product of effort directed towards organisational objectives.
4. The average human being learns under proper conditions not only to accept, but to seek
responsibility. Avoidance of responsibility, lack of ambition, and emphasis on security are generally
consequences of experience, not inherent human characteristics.
5. The capacity to exercise a relatively high degree of imagination, ingenuity, and creativity in the
solution of organisational problems is widely, not narrowly, distributed in the population.
The assumptions of Theory Y suggest a new approach in management. It emphasises on the
cooperative endeavour of management and employees. The attempt is to get maximum output with
minimum amount of control and direction. Generally, no conflict is visible between organisational
goals and individual goals. Thus, the attempts of employees which are in their best interests are also
in the interests of the organisation.
4. Explain the various types of leadership styles.
• The autocratic leader commands and expects compliance, is dogmatic and positive, and
leads by the ability to withhold or give rewards and punishment.
• The democratic, or participative leader consults with subordinates on proposed actions and
decisions and encourages participation from them. This type of leader ranges from the person
who does not take
action without subordinates' concurrence to the one who makes decisions but consults with
subordinates before doing so.
• The free-rein leader uses his or her power very little, if at all, giving subordinates a high
degree of independence in their operations. Such leaders depend largely on subordinates to
set their own goals and the means of achieving them, and they see their role as one of aiding
the operation of followers by furnishing them with information and acting primarily as a
contact with the group's external environment.
5. What are the barriers in communication? (May 2013)
A. Semantic barriers:
1. Words/symbols with different meanings. Communication symbols usually have a variety of
meanings. For example, the English word ‘round’ has 110 different meanings. The 500 most
common English words have an average of 28 definitions each. Moreover, a particular word may
have different meanings in different languages. Similarly, non-verbal symbols may convey different
meanings to different persons. In such situations, the receiver may interpret differently and
communication breaks down.
MG2351 Principles of Management

Department of ECE

2013 - 2014

2. Badly expressed messages. Poorly chosen or empty words/phrases, careless omission, lack of
coherence, bad organization of ideas, awkward sentence structure, inadequate vocabulary are some
common faults that may distort a message.
3. Faulty translations. The message must be put into words appropriate to the framework in which
the receiver operates. This requires a high level of linguistic capability. Approximate understanding
of words and their faulty translation lead to poor communication.
4. Different backgrounds. One reason for distortion of meaning is that different individuals often
interpret the same communication differently; each individual uses his own frame of reference. This
frame of reference is based on particular experience and knowledge. Thus, when people with
different knowledge and experiences try to communicate, they often have trouble getting their
meanings across.
5. In-group language (Jargon). Often, occupational or social groups develop their own terminology
or in-group language. This special language, though providing a means for precise and quick
communication within the group, creates severe communication breakdown when outsiders or other
groups are involved.
B. Psychological barriers:
6. Premature evaluation: It is the tendency of prematurely judging communication instead of
receiving it with an open mind. This may cause misunderstanding in the mind of the receiver as well
as de-motivate the sender.
6. Inattention. Another common barrier is that many receivers simply do not pay attention to the
message. One reason people do not pay attention is selective listening. Selective listening results
from a common tendency to block out information that conflicts with what we believe. When we
listen to a speech or read a newspaper, we generally pay attention only to those things that confirm
our beliefs. Sometimes people do not pay attention to communication because they are victims of
communication overload or because the information is unsolicited.
7. Poor retention. Studies show that employees retain only 50 per cent of communicated
information.
8. Distrust, threat or fear of communicator. Distrust, threat and fear undermine communication. In
a climate containing these forces, any message will be viewed with skepticism. Distrust can be due to
inconsistent behavior by the superior. Faced with threats – real or imagined – people tend to tighten
up, become defensive, and distort information.
C. Organizational barriers:
9. Faulty organization. In a large-scale enterprise where the chain of command is too long or the
span of control too big, communication will be poor. This is because successive transmissions of the
same message are decreasingly accurate. In oral communication, around 30 percent of the
information is lost in each transmission.
10. Filtering: Filtering refers to intentionally withholding or deliberately manipulating information
by the sender, either because the sender believes that the receiver does not need all the information or
that the receiver is better off not knowing all aspects of the situation. It could also be that the receiver
is simply told what he wants to hear.
11. Organizational policy: If the policy is not supportive to the flow of communication in different
directions, communication flow would not be smooth and adequate.
MG2351 Principles of Management

Department of ECE

2013 - 2014

12. Organizational rules and regulations: These affect the flow of communication by prescribing
the subject-matters to be communicated and also the channel through which these are to be
communicated.
13. Status relationships: The placing of people in superior/subordinate capacity in the formal
organization structure blocks the flow of communication, more so in the upward direction.
6. Explain trait approaches to leadership.
Trait is defined as a relatively enduring quality of an individual. The trait approach seeks to
determine ‘what makes a successful leader’ from the leader’s personal characteristics.
Prior to 1950, studies of leadership were based largely on an attempt to identify the traits that leaders
possess. Starting with the "great man" theory that leaders are born and not made, a belief dating back
to the ancient Greeks and Romans, researchers have tried to identify the physical, mental, and
personality traits of various leaders.
The major traits identified by researchers in the course of their leadership studies include the
following: physical factors such as height, weight, physique, energy, health, appearance; mental
qualities such as intelligence, alertness, administrative ability; personality characteristics such as
self-confidence, initiative, persistence, ambition, cheerfulness, enthusiasm, decisiveness; and social
characteristics such as sociability, adaptability, dominance, aggressiveness, etc.
These various traits can be classified as innate and acquired traits. Innate traits are those which are
possessed by individuals since their birth. These traits are natural and God-gifted. Acquired traits
are those which can be acquired or increased through learning, imitating, training, and other such
processes.
7. Explain various methods of communication. (Nov 2012)
Communication can be classified into different types:
• Formal communication
• Informal communication
• Downward communication
• Upward communication
• Horizontal communication
• Diagonal communication
• Oral communication
• Written communication
• Electronic communication, and
• Non-verbal communication.
8. List out the various communication networks in an organization.
I. Formal channel
• Single chain
• Wheel
• Circular
• Free flow
• Inverted V
• Single channel Vs multiple channels
II Informal chain or grapevine
MG2351 Principles of Management

Department of ECE

2013 - 2014

9. What are the computer – based communications in the modern business era?
• Electronic mail
• Internet and extranet links
• Videoconferencing
(i)
One – way video and two – way audio
(ii)
Two – way video and audio
10. Explain the factors determining effective supervision.
• Leadership
• Closeness of supervision
• Employee – orientation or human relations
• Group cohesiveness
• Delegation
• Other factors
UNIT - V
Part A
1. What is control? (May 2010)
Control is the measurement and correction of performance in order to make sure the enterprise
objectives and the plans devised to attain them are accomplished..
2. What is concurrent control? (May 2011)
This control measures for taking corrective action while any programme meet any obstacle in
this activities.
3. State the difference between feedback and feed forward control technique?
S. No.
1.

Feedback
Feed forward
It measures only the output of the process It measures the input of the process

2.

It is submissive approach

It is aggressive approach

3.

Less benefit

More benefit

4. Define Budget? (Dec 2011)
A Budget the expression of a firm’s plan is financial form for a period of time in to the
future.
5. Define budgetary control? (May 2010)
A system which uses budgets as a means of planning and controlling all aspects of producing
and selling commodities and services.
6. Write the objectives of budgetary control?
• It aims at maximization of profits
• To plan and control the income and expenditure of the organization
• To provide adequate working capital
7. What are classification of budget?
• Functional classification- Sales, production, cash, capital and master budget
• Time classification- Short, current and long term budget
• Activity level- Fixed and flexible budget
8. What is zero base budget? (Dec 2009)
MG2351 Principles of Management

Department of ECE

2013 - 2014

Initially the budget is designed from a zero base. The main element is ZBB is future objective
orientation.
9. What are the difference between PERT and CPM?
S. No.
CPM
PERT
1.
It is activity oriented
PERT is event oriented
2.

CPM is planning device

It is control device

3.

It estimates only one time

It estimates three times

4.

It is a deterministic model

It is probabilistic model

10. Define MIS. (Dec 2011)
MIS is more advance technology for solving its basic requirements. MIS used for decision
making in the various functional areas of business.
11. Define productivity.
Productivity is a measure of how much input is required to produce a given output.
12. Define OR.
OR is a systematic analysis of a problem through scientific methods, carried out by
appropriate specialists, working together as a team, finding an optimum and the most appropriate
solution to meet the given objective under a given set of constraints.
13. Define Linear Programming?
It is a mathematical technique in operation research and a plan of action solve a given problem
involving linearly related variables in order to achieve the laid down objectives in the form of
minimizing or maximizing the objective function under given set of constraints.
14. What is Inventory Control?
Inventory control refers to the control of raw materials and purchased material in
store and regulation of investment in them.
15. What is JIT? (May 2007)
Just in time inventory system, in this method the suppliers delivers the materials to the
production spot just in time to be assembled. This method reduces the cost of iventory.
16. What are objectives of value engineering?
a) It is a special type of cost reduction technique.
b) Modify and improve product design
c) Reduce the product cost
d) Increase the profit
e) Simplify the product
17. What is MNC? (May 2008)
Multinational corporation is an enterprise which own or control production or service
facilities outside the country in which they are based.
18. Write some advantages of MNCs? (Dec 2009)
• It can promote quality product at low price
• MNC leads to increase in production aggregate employment, exports and
imports of the required inputs
• It increases the government revenues.
19. What is work simplification?
It is the process of obtaining the participation of workers in simplifying their work through
time study, motion study, work flow analysis and layout of work situation.
MG2351 Principles of Management

Department of ECE

2013 - 2014

20. Define quality circles?
Quality circles are groups of people, from the same organizational area, who meet regularly
to solve problems they experience at work. Members are trained in solving problems, in
applying statistical quality control and working in groups.
Part B
1. What is the importance of controlling?
• Policy verification
• Adjustments in operations
• Measuring performance
• Comparing actual with standards
• Finding out deviations
• Correction of deviation
2. List out the critical types of critical point standards.
• Physical standards
• Cost standards
• Capital standards
• Revenue standards
• Program standards
• Intangible standards
• Goals as standards
• Strategic control.
3. What are the various budgetary control techniques? (May 2013)
I. Traditional techniques
• Personal observation
• Break – even analysis
• Statistical reports
• Budgetary control
II. Modern techniques
• Management audit
• Return on investment
• PERT and CPM
• Management information systems
4. What are the factors affecting control?
• Technology
• Human resources
MG2351 Principles of Management
•
•
•
•
•
•
•
•

Department of ECE

Government policy
Machinery and equipment
Skill of the worker
Materials
Plant equipment
Land and buildings
Capital
Research and development.

5. Explain the role of operations research in business and management.
I.
Production management
• Allocation of resources
• Project scheduling
• Inventory policy
• Equipment replacement and maintenance
II. Finance management
• Fund flow analysis
• Credit policies
• Capital requirement
III. Purchase and procurement
• Rules for purchasing
• Determining the quantity
IV. Distribution
• Location of warehouses
• Side of the warehouses
• Transportation strategies
V. Marketing management
•
•
•

Product selection
Competitive strategies
Advertising strategies.

6. What are the steps involved in planning and control?
• Routing
• Scheduling
• Dispatching
• Inspection
• Follow – up.
7. What is the importance of financial statements?

2013 - 2014
MG2351 Principles of Management
•
•
•
•
•
•

Department of ECE

2013 - 2014

Management
Shareholders
Creditors
Labors
Public
Government

8. What are the steps involved in standard purchasing procedure? (N0v 2012)
• Processing the requisition
• Location and choice of suppliers
• Placing of orders
• Follow up and expediting
• Invoice and clearance
• Maintenance of records.
9. Explain the various methods of purchasing.
• Purchasing according to the requirement
• Price forecasting method
• Purchasing for some definite future period
• Market purchasing
• Speculative purchasing
• Contract purchasing
• Scheduled purchasing
• Public buying
• Tender purchasing.
10. What are the steps in quality control?
• Fixing the quality standards
• Evaluation of measurement of quality
• Comparing the measured quality with the standard quality
• Finding out the deviation
• Reasons for variation
• Taking corrective action.

***********************************

Weitere ähnliche Inhalte

Was ist angesagt?

Concept of industrial management
Concept of industrial managementConcept of industrial management
Concept of industrial managementSanyog Kumar
 
Human resource management
Human resource managementHuman resource management
Human resource managementAnuj Bhatia
 
Management and entrepreneurship - introduction
Management and entrepreneurship - introductionManagement and entrepreneurship - introduction
Management and entrepreneurship - introductionRajendra Prasad
 
Principles of management Unit-1 BBA
Principles of management Unit-1 BBA Principles of management Unit-1 BBA
Principles of management Unit-1 BBA marudhurimaha
 
MG8591 Principles of Management
MG8591 Principles of Management MG8591 Principles of Management
MG8591 Principles of Management DineshBabuN4
 
Unit i principles of management
Unit i principles of managementUnit i principles of management
Unit i principles of managementtaruian
 
Managers and Development
Managers and DevelopmentManagers and Development
Managers and DevelopmentJameson Pagud
 
Industrial management
Industrial management Industrial management
Industrial management Anshu Singh
 
https://www.slideshare.net/
https://www.slideshare.net/https://www.slideshare.net/
https://www.slideshare.net/Joyce Wong
 
Introduction to management, functions of management, levels of management
Introduction to management, functions of management, levels of managementIntroduction to management, functions of management, levels of management
Introduction to management, functions of management, levels of managementAbhishikthSandeep1
 
Industrial management
Industrial managementIndustrial management
Industrial managementadnanqayum
 
Principles & Practice of Management - Nature - Arts, Science
Principles & Practice of Management - Nature - Arts, SciencePrinciples & Practice of Management - Nature - Arts, Science
Principles & Practice of Management - Nature - Arts, Scienceuma reur
 
02 Management notes for mba
02 Management notes for mba02 Management notes for mba
02 Management notes for mbaAwais Qasim
 
Semester 1 PRINCIPLES OF MANAGEMENT Chapter 2 PLANNING
Semester 1 PRINCIPLES OF MANAGEMENT  Chapter 2 PLANNINGSemester 1 PRINCIPLES OF MANAGEMENT  Chapter 2 PLANNING
Semester 1 PRINCIPLES OF MANAGEMENT Chapter 2 PLANNINGMAHUA MUKHERJEE
 
Industrial management
Industrial managementIndustrial management
Industrial managementAkshay Yawale
 
Management At A Glance
Management At A GlanceManagement At A Glance
Management At A Glanceguest970121
 
Solved q&a nov06
Solved q&a nov06Solved q&a nov06
Solved q&a nov06Bharath Slm
 

Was ist angesagt? (20)

Concept of industrial management
Concept of industrial managementConcept of industrial management
Concept of industrial management
 
Human resource management
Human resource managementHuman resource management
Human resource management
 
Management and entrepreneurship - introduction
Management and entrepreneurship - introductionManagement and entrepreneurship - introduction
Management and entrepreneurship - introduction
 
Principles of management Unit-1 BBA
Principles of management Unit-1 BBA Principles of management Unit-1 BBA
Principles of management Unit-1 BBA
 
MG8591 Principles of Management
MG8591 Principles of Management MG8591 Principles of Management
MG8591 Principles of Management
 
Unit i principles of management
Unit i principles of managementUnit i principles of management
Unit i principles of management
 
Managers and Development
Managers and DevelopmentManagers and Development
Managers and Development
 
Industrial management
Industrial management Industrial management
Industrial management
 
Planning notes
Planning notesPlanning notes
Planning notes
 
https://www.slideshare.net/
https://www.slideshare.net/https://www.slideshare.net/
https://www.slideshare.net/
 
Introduction to management, functions of management, levels of management
Introduction to management, functions of management, levels of managementIntroduction to management, functions of management, levels of management
Introduction to management, functions of management, levels of management
 
Principles Of Management
Principles Of ManagementPrinciples Of Management
Principles Of Management
 
Industrial management
Industrial managementIndustrial management
Industrial management
 
Principles & Practice of Management - Nature - Arts, Science
Principles & Practice of Management - Nature - Arts, SciencePrinciples & Practice of Management - Nature - Arts, Science
Principles & Practice of Management - Nature - Arts, Science
 
02 Management notes for mba
02 Management notes for mba02 Management notes for mba
02 Management notes for mba
 
Semester 1 PRINCIPLES OF MANAGEMENT Chapter 2 PLANNING
Semester 1 PRINCIPLES OF MANAGEMENT  Chapter 2 PLANNINGSemester 1 PRINCIPLES OF MANAGEMENT  Chapter 2 PLANNING
Semester 1 PRINCIPLES OF MANAGEMENT Chapter 2 PLANNING
 
Industrial management
Industrial managementIndustrial management
Industrial management
 
Management At A Glance
Management At A GlanceManagement At A Glance
Management At A Glance
 
Introduction of Industrial Management
Introduction of Industrial ManagementIntroduction of Industrial Management
Introduction of Industrial Management
 
Solved q&a nov06
Solved q&a nov06Solved q&a nov06
Solved q&a nov06
 

Ähnlich wie Pom

Principles-of-Management-Professional-Ethics.pdf
Principles-of-Management-Professional-Ethics.pdfPrinciples-of-Management-Professional-Ethics.pdf
Principles-of-Management-Professional-Ethics.pdfssuser7ec1b3
 
Principles-of-Management-Professional-Ethics.pdf
Principles-of-Management-Professional-Ethics.pdfPrinciples-of-Management-Professional-Ethics.pdf
Principles-of-Management-Professional-Ethics.pdfssuser7ec1b3
 
Principles of Management
Principles of Management Principles of Management
Principles of Management vivekkatare
 
Concept, nature & purpose of management
Concept, nature & purpose of managementConcept, nature & purpose of management
Concept, nature & purpose of managementRobin Gulati
 
Ankit yadav class 12 presentation for office 2007
Ankit yadav class 12 presentation for office 2007Ankit yadav class 12 presentation for office 2007
Ankit yadav class 12 presentation for office 2007Rao Dhruv
 
Principle Of Management -Prabhaharan429
Principle Of Management -Prabhaharan429Principle Of Management -Prabhaharan429
Principle Of Management -Prabhaharan429PRABHAHARAN429
 
Principles of Management
Principles of ManagementPrinciples of Management
Principles of Managementadwinshines
 
Management And Functions Of Management
Management And Functions Of ManagementManagement And Functions Of Management
Management And Functions Of ManagementGina Buck
 
Principles of management full notes (2)
Principles of management full notes (2)Principles of management full notes (2)
Principles of management full notes (2)DrRAJASSRMVEC
 
Mananagement PPT.pptx Diploma in Electrical engineering
Mananagement PPT.pptx Diploma in Electrical engineeringMananagement PPT.pptx Diploma in Electrical engineering
Mananagement PPT.pptx Diploma in Electrical engineering1303EEVirajAlim
 
Healthcare management
Healthcare management Healthcare management
Healthcare management Moheb Faqiri
 
Hotel administration
Hotel administrationHotel administration
Hotel administrationConfidential
 
Hotel administration
Hotel administrationHotel administration
Hotel administrationConfidential
 
Principles of management
Principles of managementPrinciples of management
Principles of managementAkhil Kirty
 

Ähnlich wie Pom (20)

Principles-of-Management-Professional-Ethics.pdf
Principles-of-Management-Professional-Ethics.pdfPrinciples-of-Management-Professional-Ethics.pdf
Principles-of-Management-Professional-Ethics.pdf
 
Principles-of-Management-Professional-Ethics.pdf
Principles-of-Management-Professional-Ethics.pdfPrinciples-of-Management-Professional-Ethics.pdf
Principles-of-Management-Professional-Ethics.pdf
 
CH 1 introduction.pptx
CH 1 introduction.pptxCH 1 introduction.pptx
CH 1 introduction.pptx
 
intro.mgt ppt.pptx
intro.mgt ppt.pptxintro.mgt ppt.pptx
intro.mgt ppt.pptx
 
UNIT 1.pptx
UNIT 1.pptxUNIT 1.pptx
UNIT 1.pptx
 
Principles of Management
Principles of Management Principles of Management
Principles of Management
 
Concept, nature & purpose of management
Concept, nature & purpose of managementConcept, nature & purpose of management
Concept, nature & purpose of management
 
Ankit yadav class 12 presentation for office 2007
Ankit yadav class 12 presentation for office 2007Ankit yadav class 12 presentation for office 2007
Ankit yadav class 12 presentation for office 2007
 
Principle Of Management -Prabhaharan429
Principle Of Management -Prabhaharan429Principle Of Management -Prabhaharan429
Principle Of Management -Prabhaharan429
 
Principles of Management
Principles of ManagementPrinciples of Management
Principles of Management
 
Management And Functions Of Management
Management And Functions Of ManagementManagement And Functions Of Management
Management And Functions Of Management
 
Henry fayol
Henry fayolHenry fayol
Henry fayol
 
Principles of management full notes (2)
Principles of management full notes (2)Principles of management full notes (2)
Principles of management full notes (2)
 
Management Concepts
Management Concepts Management Concepts
Management Concepts
 
Mananagement PPT.pptx Diploma in Electrical engineering
Mananagement PPT.pptx Diploma in Electrical engineeringMananagement PPT.pptx Diploma in Electrical engineering
Mananagement PPT.pptx Diploma in Electrical engineering
 
Healthcare management
Healthcare management Healthcare management
Healthcare management
 
Concept of management
Concept of managementConcept of management
Concept of management
 
Hotel administration
Hotel administrationHotel administration
Hotel administration
 
Hotel administration
Hotel administrationHotel administration
Hotel administration
 
Principles of management
Principles of managementPrinciples of management
Principles of management
 

Kürzlich hochgeladen

Business Model Canvas (BMC)- A new venture concept
Business Model Canvas (BMC)-  A new venture conceptBusiness Model Canvas (BMC)-  A new venture concept
Business Model Canvas (BMC)- A new venture conceptP&CO
 
FULL ENJOY Call Girls In Majnu Ka Tilla, Delhi Contact Us 8377877756
FULL ENJOY Call Girls In Majnu Ka Tilla, Delhi Contact Us 8377877756FULL ENJOY Call Girls In Majnu Ka Tilla, Delhi Contact Us 8377877756
FULL ENJOY Call Girls In Majnu Ka Tilla, Delhi Contact Us 8377877756dollysharma2066
 
Organizational Transformation Lead with Culture
Organizational Transformation Lead with CultureOrganizational Transformation Lead with Culture
Organizational Transformation Lead with CultureSeta Wicaksana
 
Falcon's Invoice Discounting: Your Path to Prosperity
Falcon's Invoice Discounting: Your Path to ProsperityFalcon's Invoice Discounting: Your Path to Prosperity
Falcon's Invoice Discounting: Your Path to Prosperityhemanthkumar470700
 
How to Get Started in Social Media for Art League City
How to Get Started in Social Media for Art League CityHow to Get Started in Social Media for Art League City
How to Get Started in Social Media for Art League CityEric T. Tung
 
Chandigarh Escorts Service 📞8868886958📞 Just📲 Call Nihal Chandigarh Call Girl...
Chandigarh Escorts Service 📞8868886958📞 Just📲 Call Nihal Chandigarh Call Girl...Chandigarh Escorts Service 📞8868886958📞 Just📲 Call Nihal Chandigarh Call Girl...
Chandigarh Escorts Service 📞8868886958📞 Just📲 Call Nihal Chandigarh Call Girl...Sheetaleventcompany
 
Mysore Call Girls 8617370543 WhatsApp Number 24x7 Best Services
Mysore Call Girls 8617370543 WhatsApp Number 24x7 Best ServicesMysore Call Girls 8617370543 WhatsApp Number 24x7 Best Services
Mysore Call Girls 8617370543 WhatsApp Number 24x7 Best ServicesDipal Arora
 
Malegaon Call Girls Service ☎ ️82500–77686 ☎️ Enjoy 24/7 Escort Service
Malegaon Call Girls Service ☎ ️82500–77686 ☎️ Enjoy 24/7 Escort ServiceMalegaon Call Girls Service ☎ ️82500–77686 ☎️ Enjoy 24/7 Escort Service
Malegaon Call Girls Service ☎ ️82500–77686 ☎️ Enjoy 24/7 Escort ServiceDamini Dixit
 
Eluru Call Girls Service ☎ ️93326-06886 ❤️‍🔥 Enjoy 24/7 Escort Service
Eluru Call Girls Service ☎ ️93326-06886 ❤️‍🔥 Enjoy 24/7 Escort ServiceEluru Call Girls Service ☎ ️93326-06886 ❤️‍🔥 Enjoy 24/7 Escort Service
Eluru Call Girls Service ☎ ️93326-06886 ❤️‍🔥 Enjoy 24/7 Escort ServiceDamini Dixit
 
Call Girls From Pari Chowk Greater Noida ❤️8448577510 ⊹Best Escorts Service I...
Call Girls From Pari Chowk Greater Noida ❤️8448577510 ⊹Best Escorts Service I...Call Girls From Pari Chowk Greater Noida ❤️8448577510 ⊹Best Escorts Service I...
Call Girls From Pari Chowk Greater Noida ❤️8448577510 ⊹Best Escorts Service I...lizamodels9
 
Call Girls Hebbal Just Call 👗 7737669865 👗 Top Class Call Girl Service Bangalore
Call Girls Hebbal Just Call 👗 7737669865 👗 Top Class Call Girl Service BangaloreCall Girls Hebbal Just Call 👗 7737669865 👗 Top Class Call Girl Service Bangalore
Call Girls Hebbal Just Call 👗 7737669865 👗 Top Class Call Girl Service Bangaloreamitlee9823
 
Call Girls Kengeri Satellite Town Just Call 👗 7737669865 👗 Top Class Call Gir...
Call Girls Kengeri Satellite Town Just Call 👗 7737669865 👗 Top Class Call Gir...Call Girls Kengeri Satellite Town Just Call 👗 7737669865 👗 Top Class Call Gir...
Call Girls Kengeri Satellite Town Just Call 👗 7737669865 👗 Top Class Call Gir...amitlee9823
 
Falcon Invoice Discounting: The best investment platform in india for investors
Falcon Invoice Discounting: The best investment platform in india for investorsFalcon Invoice Discounting: The best investment platform in india for investors
Falcon Invoice Discounting: The best investment platform in india for investorsFalcon Invoice Discounting
 
Quick Doctor In Kuwait +2773`7758`557 Kuwait Doha Qatar Dubai Abu Dhabi Sharj...
Quick Doctor In Kuwait +2773`7758`557 Kuwait Doha Qatar Dubai Abu Dhabi Sharj...Quick Doctor In Kuwait +2773`7758`557 Kuwait Doha Qatar Dubai Abu Dhabi Sharj...
Quick Doctor In Kuwait +2773`7758`557 Kuwait Doha Qatar Dubai Abu Dhabi Sharj...daisycvs
 
Uneak White's Personal Brand Exploration Presentation
Uneak White's Personal Brand Exploration PresentationUneak White's Personal Brand Exploration Presentation
Uneak White's Personal Brand Exploration Presentationuneakwhite
 
Cheap Rate Call Girls In Noida Sector 62 Metro 959961乂3876
Cheap Rate Call Girls In Noida Sector 62 Metro 959961乂3876Cheap Rate Call Girls In Noida Sector 62 Metro 959961乂3876
Cheap Rate Call Girls In Noida Sector 62 Metro 959961乂3876dlhescort
 
FULL ENJOY Call Girls In Mahipalpur Delhi Contact Us 8377877756
FULL ENJOY Call Girls In Mahipalpur Delhi Contact Us 8377877756FULL ENJOY Call Girls In Mahipalpur Delhi Contact Us 8377877756
FULL ENJOY Call Girls In Mahipalpur Delhi Contact Us 8377877756dollysharma2066
 
Katrina Personal Brand Project and portfolio 1
Katrina Personal Brand Project and portfolio 1Katrina Personal Brand Project and portfolio 1
Katrina Personal Brand Project and portfolio 1kcpayne
 

Kürzlich hochgeladen (20)

Business Model Canvas (BMC)- A new venture concept
Business Model Canvas (BMC)-  A new venture conceptBusiness Model Canvas (BMC)-  A new venture concept
Business Model Canvas (BMC)- A new venture concept
 
FULL ENJOY Call Girls In Majnu Ka Tilla, Delhi Contact Us 8377877756
FULL ENJOY Call Girls In Majnu Ka Tilla, Delhi Contact Us 8377877756FULL ENJOY Call Girls In Majnu Ka Tilla, Delhi Contact Us 8377877756
FULL ENJOY Call Girls In Majnu Ka Tilla, Delhi Contact Us 8377877756
 
Organizational Transformation Lead with Culture
Organizational Transformation Lead with CultureOrganizational Transformation Lead with Culture
Organizational Transformation Lead with Culture
 
Falcon's Invoice Discounting: Your Path to Prosperity
Falcon's Invoice Discounting: Your Path to ProsperityFalcon's Invoice Discounting: Your Path to Prosperity
Falcon's Invoice Discounting: Your Path to Prosperity
 
How to Get Started in Social Media for Art League City
How to Get Started in Social Media for Art League CityHow to Get Started in Social Media for Art League City
How to Get Started in Social Media for Art League City
 
Chandigarh Escorts Service 📞8868886958📞 Just📲 Call Nihal Chandigarh Call Girl...
Chandigarh Escorts Service 📞8868886958📞 Just📲 Call Nihal Chandigarh Call Girl...Chandigarh Escorts Service 📞8868886958📞 Just📲 Call Nihal Chandigarh Call Girl...
Chandigarh Escorts Service 📞8868886958📞 Just📲 Call Nihal Chandigarh Call Girl...
 
Mysore Call Girls 8617370543 WhatsApp Number 24x7 Best Services
Mysore Call Girls 8617370543 WhatsApp Number 24x7 Best ServicesMysore Call Girls 8617370543 WhatsApp Number 24x7 Best Services
Mysore Call Girls 8617370543 WhatsApp Number 24x7 Best Services
 
Malegaon Call Girls Service ☎ ️82500–77686 ☎️ Enjoy 24/7 Escort Service
Malegaon Call Girls Service ☎ ️82500–77686 ☎️ Enjoy 24/7 Escort ServiceMalegaon Call Girls Service ☎ ️82500–77686 ☎️ Enjoy 24/7 Escort Service
Malegaon Call Girls Service ☎ ️82500–77686 ☎️ Enjoy 24/7 Escort Service
 
Eluru Call Girls Service ☎ ️93326-06886 ❤️‍🔥 Enjoy 24/7 Escort Service
Eluru Call Girls Service ☎ ️93326-06886 ❤️‍🔥 Enjoy 24/7 Escort ServiceEluru Call Girls Service ☎ ️93326-06886 ❤️‍🔥 Enjoy 24/7 Escort Service
Eluru Call Girls Service ☎ ️93326-06886 ❤️‍🔥 Enjoy 24/7 Escort Service
 
Call Girls From Pari Chowk Greater Noida ❤️8448577510 ⊹Best Escorts Service I...
Call Girls From Pari Chowk Greater Noida ❤️8448577510 ⊹Best Escorts Service I...Call Girls From Pari Chowk Greater Noida ❤️8448577510 ⊹Best Escorts Service I...
Call Girls From Pari Chowk Greater Noida ❤️8448577510 ⊹Best Escorts Service I...
 
Falcon Invoice Discounting platform in india
Falcon Invoice Discounting platform in indiaFalcon Invoice Discounting platform in india
Falcon Invoice Discounting platform in india
 
Call Girls Hebbal Just Call 👗 7737669865 👗 Top Class Call Girl Service Bangalore
Call Girls Hebbal Just Call 👗 7737669865 👗 Top Class Call Girl Service BangaloreCall Girls Hebbal Just Call 👗 7737669865 👗 Top Class Call Girl Service Bangalore
Call Girls Hebbal Just Call 👗 7737669865 👗 Top Class Call Girl Service Bangalore
 
Call Girls Kengeri Satellite Town Just Call 👗 7737669865 👗 Top Class Call Gir...
Call Girls Kengeri Satellite Town Just Call 👗 7737669865 👗 Top Class Call Gir...Call Girls Kengeri Satellite Town Just Call 👗 7737669865 👗 Top Class Call Gir...
Call Girls Kengeri Satellite Town Just Call 👗 7737669865 👗 Top Class Call Gir...
 
Falcon Invoice Discounting: The best investment platform in india for investors
Falcon Invoice Discounting: The best investment platform in india for investorsFalcon Invoice Discounting: The best investment platform in india for investors
Falcon Invoice Discounting: The best investment platform in india for investors
 
Quick Doctor In Kuwait +2773`7758`557 Kuwait Doha Qatar Dubai Abu Dhabi Sharj...
Quick Doctor In Kuwait +2773`7758`557 Kuwait Doha Qatar Dubai Abu Dhabi Sharj...Quick Doctor In Kuwait +2773`7758`557 Kuwait Doha Qatar Dubai Abu Dhabi Sharj...
Quick Doctor In Kuwait +2773`7758`557 Kuwait Doha Qatar Dubai Abu Dhabi Sharj...
 
Uneak White's Personal Brand Exploration Presentation
Uneak White's Personal Brand Exploration PresentationUneak White's Personal Brand Exploration Presentation
Uneak White's Personal Brand Exploration Presentation
 
Cheap Rate Call Girls In Noida Sector 62 Metro 959961乂3876
Cheap Rate Call Girls In Noida Sector 62 Metro 959961乂3876Cheap Rate Call Girls In Noida Sector 62 Metro 959961乂3876
Cheap Rate Call Girls In Noida Sector 62 Metro 959961乂3876
 
FULL ENJOY Call Girls In Mahipalpur Delhi Contact Us 8377877756
FULL ENJOY Call Girls In Mahipalpur Delhi Contact Us 8377877756FULL ENJOY Call Girls In Mahipalpur Delhi Contact Us 8377877756
FULL ENJOY Call Girls In Mahipalpur Delhi Contact Us 8377877756
 
Katrina Personal Brand Project and portfolio 1
Katrina Personal Brand Project and portfolio 1Katrina Personal Brand Project and portfolio 1
Katrina Personal Brand Project and portfolio 1
 
(Anamika) VIP Call Girls Napur Call Now 8617697112 Napur Escorts 24x7
(Anamika) VIP Call Girls Napur Call Now 8617697112 Napur Escorts 24x7(Anamika) VIP Call Girls Napur Call Now 8617697112 Napur Escorts 24x7
(Anamika) VIP Call Girls Napur Call Now 8617697112 Napur Escorts 24x7
 

Pom

  • 1. MG2351 Principles of Management Department of ECE 2013 - 2014 UNIT – I Part A 1. Define Management. (April 2008) Management is the process of designing and maintaining an environment in which individuals, working together in groups, accomplish their aims effectively and efficiently. According to Koontz and Weilhrich "Management is process of designing and maintaining of an environment in which individuals working together in groups, efficiently and efficiently attain the organizational goals." 2. State the Functions of Manager. (May 2010) P – Planning O – Organising S – Staffing D – Directing CO – Coordinating R – Reporting B – Budgeting 3. What are the various skills required by a manager? Managers require three kinds of skills Technical Skill – Knowledge of and proficiency in working with tools and technology Human Skill – Ability to work with people Conceptual Skill – Ability to recognize important element in a situation and understand relationship among elements 4. Write some characteristics of Management Management is a continuous process. 2) Managers use the resources of the organisation both physical as well as human to achieve the goals. 3) Management aims at achieving the organisation goals by ensuring effective use of resources. 5. Define Productivity. (May 2009) It implies effectiveness and efficiency in individual and organizational performance. Productivity = Outputs Inputs (within a time period, quality considered) 6. What is Effectiveness & Efficiency ? (Dec 2010) Effectiveness is the achievement of objectives and Efficiency is the achievement of the ends with the least amount of resources. Effectiveness is doing right things & efficiency is doing things rightly. 7. What are management levels? Top level management. 2) Middle level management. 3)Lower level management. 8. Write some important functions of top level management. (May 2007) • To formulate goals and policies of the company. • Formulate budgets. • To appoint top executives. 9. What is social responsibility? Social responsibility is the part of the management to initiate actions to protect the interest of the society. 10. Write Fayol's fourteen principles of management. 1) Division of work. 2) Authority and Responsibility. 3) Discipline. 4) Unity of command. 5) Unity of direction. 6) Subordination of Individual interest to general interest. 7) Remuneration. 8) Centralisation 9) Scalar chain.
  • 2. MG2351 Principles of Management Department of ECE 2013 - 2014 10) Order. 11) Equity. 12) Stability of Tenure. 13) Initiative. 14) Esprit de Corps. 11. What is scalar chain? The instructions and orders should be sent from the top management to the lower management. 12. Explain: Management is both —A science and an art. Management is a science because it contains general principles. It is also an art because it requires certain personal skills to achieve desired results. 13. What is Esprit-de-corps? It means ‘Unity is strength’. In an organization, amongst the employees there should be harmony and unity. 14. List out the claimants of the business enterprise. 1) Shareholders. 2) Employees. 3) Customers. 4) Creditors. 5) Suppliers. 6) Government 15. What are the major contributions of Taylor? The major contributions of F.W.Taylor are as follows • He developed the principle of division of labor/work • He developed method study • He advocated time study • He developed certain principles to breakup each job into small independent elements • He developed the concept of fair day’s work • He proposed the functional organization 16. Write some important functions of top level management. (May 2009) To formulate goals and policies of the company. 2) formulate budgets. 3) To appoint top executives. 17. What is time study? Time study refers to the measuring of (under controlled conditions) the time for completing a particular process using specific materials and techniques. This will be useful in arriving at a scientific standard for each work task in the marketing process. 18. Define partnership Under the Indian Partnership Act of 1932, Partnership is defined as –“the relation between persons who have agreed to share the profits of a business carried on by all or by any one of them acting for all”. 19. What do you mean by ‘unity of command’? (Dec 2010) ‘Unity of command’ is an important principle proposed by Henry Fayol. This implies that an employee should receive orders from one superior only. 20. What is centralization? The organization is centralized when the power is concentrated in the hands of few people. Part B 1. Explain the Functions of Management. Planning Planning is the function that determines in advance what should be done. It is looking ahead and preparing for the future. It is a process of deciding the business objectives and charting out the methods of attaining those objectives. In other words, it is the determination of what is to be done, how and where it is to be done, who is to do it and how results are to be evaluated. This is done not only for the organisation as a whole but for every division, department or sub-:unit of the organisation. Thus, planning is a function which is performed by managers at all levels—top, middle and supervisory. Plans made by top management for
  • 3. MG2351 Principles of Management Department of ECE 2013 - 2014 the organisation as a whole may cover periods as long as five or ten years. Plans made by middle or first line managers, cover much shorter periods. Such plans may be for the next day's work, for example, or for a two-hour meeting to take place in a week. Organising To organise a business is to provide it with everything useful to its functioning: personnel, raw materials, tools, capital. All this may be divided into two main sections, the human organisation and the material organisation. Once managers have established objectives and developed plans to achieve them, they must design and develop a human organisation that will be able to carry out those plans successfully. According to Alien, this organisation refers to the "structure which results from identifying and grouping work, defining and delegating responsibility and authority, and establishing relationships." Staffing may also be considered an important function involved in building the human organisation. In staffing, the manager attempts to find the right person for each job. Staffing fixes a manager's responsibility to recruit and to make certain that there is enough manpower available to fill the various positions needed in the organisation. Staffing involves the selection and training of future managers and a suitable system of compensation. Staffing obviously cannot be done once and for all, since people are continually leaving, getting fired, retiring and dying. Often too, the changes in the organisation create new positions, and these must be filled. The other aspect of organizing involves material organization – organizing other inputs like materials, equipment, facilities, power, information systems, finance, etc. for effective running of the organization. Different objectives require different kinds of organisation to achieve them. For example, an organisation for scientific research will have to be very different from one for manufacturing bicycles. Producing bicycles requires assembly-lint techniques, whereas scientific research requires teams of scientists and experts in various disciplines. Such people cannot be organised on an assembly-line basis. Directing After plans have been made and the organisation has been established and staffed, the next step is to move towards its defined objectives. This function can be called by various names: 'leading’, 'directing, 'motivating", 'actuating’, and so on. But whatever the name used to identify it, in carrying out this function the manager explains to his people what they have to do and helps them do it to the best of their ability. Directing thus involves three sub-functions—communication, leadership and motivation. Communication is the process of passing information and understanding from one person to another. Leadership is the process by which a manager guides and influences the work of his subordinates. Motivation means arousing desire in the minds of workers to give their best to the enterprise. It is the act of stimulating or inspiring workers. If the workers of an enterprise are properly motivated they will pull their weight effectively, give their loyalty to the enterprise, and carry out their task effectively. Two broad categories of motivation are: financial and non-financial. Financial motivation takes the form of salary, bonus, profitsharing, etc., while non-financial motivation takes the form of job security, opportunity of advancement, recognition, praise, etc. Controlling The manager must ensure that everything occurs in conformity with the plans adopted, the instructions issued and the principles established. This is the controlling function of management, and involves three elements. 1. Establishing standards of performance. 2. Measuring current performance and comparing it against the established standards. 3. Taking action to correct any performance that does not meet those standards.
  • 4. MG2351 Principles of Management Department of ECE 2013 - 2014 In the absence of sound control, there is no guarantee that the objectives which have been set will be realised. The management may go on committing mistakes without knowing them. Control compels events to conform to plans. 2. Discuss whether Management is a Science or Art. Professional managers need to adopt a rational, scientific approach in dealing with the concrete, measurable aspects of business such as setting objectives/goals, formulating action plans, efficiently organizing required resources, and devising controls. In this respect, management is a science. On the other hand, managers have also to deal with the human aspects of the organization – providing leadership, motivation, guidance to individuals and groups, and resolving conflicts. In this respect, management is an art. 3. Explain the Henry Fayol’s principles of management? (April 2009) Give brief explanations of Fayol’s 14 principles listed below: 1) Division of work. 2) Authority and responsibility 3) Discipline 4) Unity of command. 5) Unity of direction. 6) Subordination of individual interest to general interest. 7) Remuneration. 8) Centralization 9) Scalar chain 10) Order 11) Equity 12) Stability of Tenure 13) Initiative and 14) Esprit de Corps. 4. Explain the contributions of FW Taylor. Explain the salient features of Scientific Management, Time and Motion Study, Differential Payment system, Reorganization of Supervision, Scientific Recruitment and Training of Workers, and Cooperation between Management and Workers proposed by Taylor. 5. What are the managerial skills required at different levels of management? (May 2013) Skills needed by managers: To be successful, managers need four kinds of skills, though the relative importance of the skills differ at various levels of management. [i] Technical skill: Knowledge, proficiency in activities involving methods, processes, procedures. [ii] Human skill: Ability to work with, interact with people. [iii] Conceptual skill: Ability to see the ‘big picture’, to understand the importance and relationships among elements. [iv] Design skill: Ability to solve problems. 6. Explain the importance of management? How Management is differentiated from administration While numerous definitions of the term exist in management literature, one convenient definition that meets our purpose would be: “the process of planning, organizing, directing and controlling human efforts to achieve organizational objectives”. Importance: Management is essential in all organized effort, be it a business activity or any other activity. It is required for all types of organizations – business and non-business, for-profit and notfor-profit, manufacturing and services. Ever since people began forming groups to accomplish aims they could not achieve as individuals, managing has been essential for coordinating individual
  • 5. MG2351 Principles of Management Department of ECE 2013 - 2014 efforts. In a competitive economy, the quality and performance of management determine the success of an organization; indeed, they determine its very survival. 7. Explain the Weber’s ideal bureaucracy. • Specialization of laws • Formal rules and procedures • Impersonality • Well defined hierarchy • Career advanced based on merit 8. Environmental factors: (May2013) • Political and legal • Economy • Competition • Technology • Socio – cultural • Natural environment 9. Explain the role of managers in detail. (Nov 2012) Interpersonal roles: • Figurehead: Duties of a ceremonial nature such as inaugurating functions. • Leader: Motivate, encourage, guide employees. • Liaison: Interact with other departments, divisions, outsiders. II Informational roles: • Recipient: Receive information about activities. • Disseminator: Pass information to subordinates. • Spokesman: Transmit information to outsiders. III Decision roles: • Entrepreneur: Seek, develop ideas for new products, markets, technology, etc. • Disturbance handler: Solving problems. • Resource allocator: Allocate work, resources, delegate authority. • Negotiator: Negotiate with clients, suppliers, unions, etc. 10. What are the trends and challenges of management? • Workforce diversity • Ethics
  • 6. MG2351 Principles of Management • • • • • • • Department of ECE 2013 - 2014 Innovation and change Total Quality Mgt Re-engineering Empowerment and teams Bimodal workforce Downsizing Contingent workers UNIT – II Part A 1. Definition of planning. (May 2008) The determination of what is to be done, how and where it is t be done, who is to do it and how results are to be evaluated. 2. Types of planning Short range and long range planning repeated and single use planning and functional planning 3. What are the Qualities of good plan? Simple , logical, flexible, practical, stable and it must be complete and integrated 4. What are Rules ? Statement of expected results expressed in quantitative terms. 5. Define Procedure It is a series of related task that up the chronological sequence and the established way of performing the work to be accomplished. 6. Define Budget. Statement of expected results expressed in quantitative terms. 7. Define Objectives. (Dec 2009) Objectives are goals established to guide the efforts of the company and each of its components 8. What is MBO? Process whereby the superior and subordinate of an organization jointly identify its common goal, define each individual’s major areas of responsibility in terms of results expected of him and use this measures as guides for operating the unit and assessing the contribution of each if its members 9. What is TOWS matrix ? It is a conceptual frame work for systematic analysis, which facilitates matching the external threats and opportunities with the internal weakness and strength of the organization. 10. Define Strategy. (Dec 2010) Determination of basic long term objectives and of courses of action and allocation of resources to achieve these aims. 11. What are Policies ? ( May 2007) Verbal, written, or implied overall guide setting up boundaries that supply the general limits and direction in which managerial action will take place. 12. State different strategies of Porter’s generic strategy. Overall cost leadership strategy, differentiation strategy and focused strategy 13. Define Planning Premises. (May 2008) Anticipated environment in which plan are expected to operate. They include assumptions or forecasts of the future and known conditions that will effect the operation of plans 14. Define MIS. (Dec 2009)
  • 7. MG2351 Principles of Management Department of ECE 2013 - 2014 MIS is more advance technology for solving its basic requirements. MIS used for decision making in the various functional areas of business. 15. Explain the terms decision-making. It is a process of selection from a set of alternative courses f action which is thought to fulfill the objectives of the decision problem more satisfactory than others. 16. What are classification of budget? a) Functional classification- Sales, production, cash, capital and master budget b) Time classification- Short c) current and long term budget d) Activity level- Fixed and flexible budget 17. What is zero base budget? Initially the budget is designed from a zero base. The main element is ZBB is future objective orientation. 18. What is DSS ? (Dec 2010) DSS –use computers to facilitate the decision making process of semi structured tasks 19. Benefits of planning. (May 2008) Emphasis on objectives, minimizes uncertainty, facilitates control, improves coordination, secure economy, encourage innovation and improve competitive strength. 20. What are the purpose of planning? • To determine the direction of an organization • To minimize wastages • To reduce the risk or uncertainty • To facilitate control. Part B 1. Explain the Steps in Planning. (Nov 2012) The basic planning process consists of the following steps: [1] Perception of opportunities, [2] Establishing objectives, [3] Making planning premises, [4] Identification of alternatives, [5] Evaluation of alternatives, [6] Choice of appropriate plan, [7] Formulation of supporting plans, and [8] Establishing action plan for implementation. 2. Explain the types of plans. (May 2013) I. II. Standing plans • Mission and purpose • Objectives • Strategies • Policies • Procedures • Rules Single use plans • Programmes • Budgets • Schedules • Methods • Projects
  • 8. MG2351 Principles of Management Department of ECE 2013 - 2014 3. Explain the Tow’s matrix with example. The letters S, W, O and T respectively stand for Strengths, Weaknesses, Opportunities and Threats. It is a conceptual framework for a systematic analysis that facilitates matching of the external opportunities and threats with the internal strengths and weaknesses of the organization. Combining of these factors may require distinct strategic choices. Internal strengths (S) Internal weaknesses (W) External opportunities SO strategy: Utilize the WO WO strategy: (O) company’s Development stra strategy to strengths to take advantage of overcome weakness opportunities. to exploit opportunities. External threats (T) ST strategy: Use strengths to WT strategy: avoid Retrenchment, liquidation, or cope with threats. or joint venture to minimize weaknesses, threats. Recently, the TOWS Matrix concept has been introduced for planning mergers, acquisitions, joint ventures and alliances. Whenever two partners consider joint activities, it is prudent to analyze the strengths and weaknesses of each partner as well as their opportunities and threats. 4. Explain the various techniques of decision making. Explain the salient features of Expected Monetary Value (EMV) criterion and Expected Opportunity Loss (EOL) criterion. Explain the salient features of [i] Maximax criterion, [ii] Maximin criterion, [iii] Minimax regret criterion, [iv] Laplace criterion, and [v] Hurwicz criterion 5. Explain the steps involved in strategy formulation and implementation process. (Nov 2012) Prof. Michael Porter has suggested that strategy formulation requires an analysis of the attractiveness of an industry and the company’s position within that industry. This analysis becomes the basis for formulating generic strategies. Industry Analysis In the analysis of the industry, Porter identified five forces: [1] the competition among companies, [2] the threat of new companies entering the market, [3] the possibility of using substitute products or services, [4] the bargaining power of the suppliers, and [5] the bargaining power of buyers or customers. On the basis of such industry analysis, a company may adopt generic strategies. These strategies are generic because they may be suitable on a broad level for different kinds of organizations. Any enterprise, however, may use more than one strategy. 1. Overall Cost Leadership Strategy: This strategic approach aims at reduction in costs, based to a great extent on experience. The emphasis may be on keeping a close watch on costs in areas such as R&D, operations, sales, and service. The objective is for a company to have a low-cost structure compared with its competitors. This strategy requires a large market share and cost- efficient operations. Example: Maruti 800 cars. 2. Differentiation Strategy: A company following this strategy attempts to offer something unique in the industry in terms of products or services. Example: Dell Computers. 3. Focused Strategy: A company adopting a focused strategy concentrates on special groups of customers, a particular product line, a specific geographic region, or other aspects that become the focal point of the firm’s efforts. Rather than serving the entire market with its products or services, the enterprise may concentrate on a specific segment of the market. A low-cost strategy,
  • 9. MG2351 Principles of Management Department of ECE 2013 - 2014 differentiation, or both may accomplish this. An example of focused low-cost strategy is Businessworld , a competitively priced business magazine. The focused differentiation strategy is exemplified by Hidesign leather products which specializes in exclusive leather items and manages to charge premium prices for them. In general, a company must to choose a generic strategy and should not “get stuck in the middle”, according to Porter. 6. Write a note on BCG Portfolio matrix. The Boston Consulting Group (abbreviated BCG) developed the business portfolio matrix. It shows the linkages between the growth rate of the business and the relative competitive position of the firm, identified by its market share. Business Hig growth h Stars Question rate marks Low Cash cows Dogs Strong Weak Competitive position Businesses in the ‘question marks’ quadrant, with a weak market share and a high growth rate, usually require cash investment so that they can become ‘stars’. ‘Stars’ are in the high-growth, strongly competitive position. These kinds of businesses have opportunities for growth and profit. The ‘cash cows’, with a strong competitive position and a low growth rate, are usually well established in the market, and such businesses are in a position to provide cash for the growth of ‘stars’ and ‘question marks’. The ‘dogs’ are businesses with a low growth rate and a weak market share. These businesses are usually not profitable and should generally be disposed off. The BCG Matrix was developed for large corporations with several divisions that are often organized as Strategic Business Units (SBUs). 7. What are the types of decisions? • • Programmed and Non-Programmed Decisions Major and Minor Decisions Degree of futurity of decision Impact of the decision on other functional areas Recurrence of decisions • • Routine and Strategic Decisions Individual and Group Decisions 8. What are the types of planning? • Corporate planning and functional planning • Strategic planning and operational planning • Long – term and short – term planning • Proactive planning and reactive planning
  • 10. MG2351 Principles of Management • Department of ECE 2013 - 2014 Formal and informal planning 9. List out the features of planning. • • • • • • • • • • Planning – a primary function Planning – a dynamic process Planning – based on objectives and policies Planning – a selective process Pervasiveness of planning Planning – an intellectual process Planning is directed towards effiency Planning – focus with future activities Flexibility of planning Planning is based on facts. 10. What are the importances of planning? • Primary of planning • To offset uncertainty and change • To focus attention on objectives • To help in coordination • To help in control • To increase organizational effectiveness. 11. What are the steps involved in MBO process? (May 2013) • Establish long-range objectives and plans consistent with the mission and philosophy of the organization. • Establish specific short-term organizational objectives covering each area such as marketing, profitability, productivity, and so on. These must be supportive of the long range objectives and plans. • Establish action plans covering individual performance objectives and standards. It is at this level that managers and subordinates work closely in setting their individual goals. Subordinates are given sufficient latitude to devise and implement strategies to achieve these objectives. • Periodically appraise results based on a jointly agreed performance appraisal methodology. • Take appropriate corrective actions if performance deviates from the objectives set. UNIT III Part A 1. Define Organizing? Organizing is the process of identifying and activities required to attain the objectives, delegating , creating responsibility and establishing relationships I people to work effectively. 2. Mention any four characteristics of an organization. a. Common objectives b. Specialisation or Division of Labour c. Authority of structure d. Group of persons
  • 11. MG2351 Principles of Management Department of ECE 2013 - 2014 3. List out the steps involved in organization process. a. Determination of activities b. Grouping of activities c. Assignment of Duties d. Delegation of authority 4. Mention the various principles involved in organization. 1. Principle of unity of objective 2. Principle of division of work or specialization 3. Principle of efficiency 4. Principle of span of control 5. Differentiate between formal and informal organization. Point of SI no. Formal organisation view Informal organisation It is created deliberately and consciously by the frames of the organisation. It is created for achieving legitimate objectives of the organisation. It is created spontaneously naturally. Nature Planned and official Unplanned and unofficial. Size It may quite large. It may be small size. 1. Origin 2. Purpose 3. 4. It is created by members of the organisation for social and psychological satisfaction. 6. Mention the three categories of relationships in span of management. a. Direct single relationship b. Direct group relationships c. Cross relationship 7. State the important factors in determining an effective span of management. 1. Capacity of superior 2. Capacity of subordinates 3. Nature of work 4. Type of technology 5. Delegation of authority 8. What are the types of departmentation ? (May 2008) 1. Departmentation by function. 2. Departmentation by Territory or Geography. 3. Departmentation by Customers. 4. Departmentation by equipment or process. 5. Departmentation by product or service. 9. State the advantages of departmentation by function. (Dec 2009) Advantages: 1. It is most logical, scientific, time proven and natural method of departmentation. 2. It provides specialization of work which makes maximum utilization of manpower and other resources. 3. It ensures proper performance control.
  • 12. MG2351 Principles of Management Department of ECE 2013 - 2014 4. It facilitates delegation of authority and therefore, reduces the burden of top executives. 10. State the disadvantages of departmentation by function. (May 2010) Disadvantages: 1. There is a tendency for overspecialization. The department managers are experts in handling the problems in their department alone. They may not be able to understand the problem of other departments. 2. Functional departmentation discourages communication across functions so that the workers develop a narrow technical point of horizontal conflicts. 3. It increases the workload and responsibility of the departmental heads. 4. It does not offer any scope for training for the overall development of manager 11. Give a note on departmentation by customers. This type of departmentation is preferred when the needs of customers are different in nature. Big organizations provide special services to different types of customer. 12. What is departmentation by product? Departmentation By Product Or Service: In this case, the units are formed according to the product. It is more useful in multi-line corporations where ] expansion and diversification, manufacturing and mark characteristics of the product are of primary concern. 13. Define authority. (May 2009) "Authority is the right to give orders and the power to exact obedience". 14. List out the sources of authority. a. Formal authority theory b. Acceptance Authority Theory c. Competence theory 15. What is line authority? Line authority is the direct authority which a superior exercises over a number of subordinates to carry out orders and instructions. In an organizing process, authority is delegated to the individuals to perform the activities. 16. What is staff authority? A staff person assists the line people in attaining their objectives. Staff authority is purely advisory. Types of staff authority are a) Advisory staff authority, b) Compulsory staff authority & c) Concurrent staff authority. 17. State the types of organizational charts. a) Vertical or Top-to-Bottom chart b) Horizontal or Left-to-right chart c) Circular or concentric chart 18. What is departmentation by process? Departmentation By Process Or Equipment: Under this type of departmentation, activities are grouped on the basis of production processes or equipment involved. 19. What are various stages of staffing Process?
  • 13. MG2351 Principles of Management Department of ECE 2013 - 2014 External environment Enterprise plans Organization plans Number and kinds of managers required External sources Recruitment Selection Placement Promotion Separation Analysis of present and future needs for managers Internal sources Appraisal Career Strategy Training &Develop ment Manager inventory 20. What is Performance Appraisal? What are the different methods/techniques of Performance appraisal? (May 2012) Performance Appraisal is evaluating the actual performance of the employee for determining the compensation and identifying the potential of the employees. Methods/Techniques of Performance Appraisal: (i) Trait-based appraisal:(job knowledge, leadership, judgment, ability, initiative, loyalty): Graphic scale method, ranking method, grading system, forced distribution method, check list method, Critical incident method, group appraisal (ii)Appraisal by results: MBO, Behaviorally Anchored Rating(BARS), Assessment Centre, 360 degree performance appraisal. Part B 1. Differentiate formal and informal organization. Formal organization Informal organization 1. It is a prescribed structure of roles and 1. It is a natural and spontaneous structure, relationships consciously created to arising out of the social tendency of people to achieve associate and interact. Management has no a common objective. role in its emergence and functioning. 2. Its values, goals, tasks are oriented Its values, goals, tasks centre around towards individual and group satisfaction, esteem, productivity, profitability, efficiency, etc. affiliation, friendship, etc. 3. It is well-defined in shape. Majority of formal organizations are pyramid-shaped. Ranks of individuals are made clear by the use of titles. 4. There is a prescribed, mostly written system of reward and punishment. Rewards can be both monetary and non- It is shapeless. There are a number of multidirectional, intricate relationships which cannot be easily charted. There is an unwritten system of reward and punishment. Rewards take the form of continued membership, social esteem,
  • 14. MG2351 Principles of Management Department of ECE monetary. 2013 - 2014 satisfaction, group leadership, etc. Punishments are isolation, censure, harassment, etc. 5. This organization is usually very This organization is not very enduring, being enduring and may grow to any size. dependent upon the sentiments of members, which often change. It also tends to remain small. 2. Discuss art of delegation of authority. (Nov 2012) Personal attitude towards delegation • Receptiveness • Willingness to let go • Willingness to let other make mistake • Willingness to trust subordinates • Willingness to establish and use broad controls Guidelines for overcoming weak delegation • Defined assignments • Selection of appropriate person • Open lines of communication • Proper control • Rewarding 3. List out the steps involved in selection process. • Issuing blank application • Initial interview • Employment test • Checking references • Physical and medical examination • Final interview 4. Explain the training methods in detail. (May 2012) On the job training method • Job rotation • Apprenticeship and coaching • Committee assignments • Experience
  • 15. MG2351 Principles of Management • Department of ECE 2013 - 2014 Temporary promotions Off – the job training method • Lecture • Conference and seminars • Role playing • Case studies • Programmed instructions • Business games • In basket method • Sensitivity training 5. What are the types of organizational structure? 1. Line organization: It is also known as scalar, military, or vertical organization. It is the oldest form of organization structure. It is most common among small companies. The authority flows in a direct line from the top management down to different levels of managers and subordinates down to the operative level of workers. Merits: 1. Simplicity, 2. Discipline, 3. Prompt decisions, 4. Orderly communication, 5. Easy supervision and control. Demerits: 1. Autocratic approach, 2. Problems of coordination, 3. Not suited for large, complex organizations. 2. Line and staff organization: A pattern in which staff specialists assist and advise line managers to perform their duties. The staff positions or departments are generally of advisory nature. The staff specialists may provide services to a particular position, department, or the organization as a whole. Merits: 1. Planned specialization, 2. Quality decisions, 3. Prospect for personal growth, 4. Training ground for personnel. Demerits: 1. Lack of well defined authority, 2. Potential for line-staff conflicts. 3. Functional organization: It is the most widely used structure in medium and large organizations making limited number of products. It is created by grouping activities on the basis of functions such as production, marketing, finance, etc. Several departments may be created by dividing the basic functions further into sub-functions, e.g. marketing into sales, distribution, advertising, etc. Staff positions may be created to assist the line functions. Merits: 1. Expertise through specialization, 2. Order and clarity, 3. Good coordination and control, 4. Economy in using resources, 5. Promotes professional achievement. Demerits: 1. Focus on departmental goals, 2. Potential for conflicts between departments. 4. Divisional organization: Growth of an organization through geographic and product diversification requires the adoption of the divisional structure. In this form, the organization is divided into several autonomous business units. Each unit is selfcontained and independent. Each division has its own manufacturing, marketing, etc. departments. The various divisions may be created on the basis of products, territories, or customers. For example, Hindustan Unilever has personal care, home care, food products, etc. divisions, each offering a related range of products. Indian Railways is organized on the basis of Northern Railway, Southern Railway, etc. based on regions. Airtel has separate divisions to cater to corporate customers and retail customers.
  • 16. MG2351 Principles of Management Department of ECE 2013 - 2014 Merits: 1. Efficient way to manage diversified activities, 2. Better focus on results, 3. Greater flexibility, 4. Better customer service. Demerits: 1. Duplication of facilities, 2. Complex control systems. 5. Project organization: It is a form of divisional organization suitable for companies which have to execute major projects. Each project is managed as a separate division which exists during the life time of the project. When a particular project is completed, the concerned division may cease to exist. Each project is headed by a project manager who coordinates the activities of the project and is responsible for the completion of the project. The staff for each project are drawn from the from various functional departments and report to the project manager. Once the project is completed, they go back to their respective departments. Examples of projects are civil construction contracts, new product development, etc. Merits: 1. Maximum use of specialized knowledge and skills, 2. Flexibility in using resources. Demerits: Problems of dual authority and responsibility. 6. Matrix organization: It is somewhat similar to project organization. The basic difference is that project organization is suitable for taking up a small number of large, long duration projects; matrix organization is suitable for taking up a large number of small, short duration projects. The activities of various such projects can be accomplished through temporary departments. Matrix organization is twodimensional in structure; the project structure and functional structure are merged together to create the matrix structure. Here also, a project manager is appointed to coordinate the activities of the project. Personnel are drawn from their respective functional departments. Upon completion of the project, these people may return to their original departments for further assignment. Thus each functional staff has two bosses – his administrative head and his project manager. Merits: 1. Maximum use of specialized knowledge and skills, 2. Flexibility in using resources. Demerits: Problems of dual authority and responsibility. 6. What are the types of departmentation? (May 2013) Departmentation based on functions Each major function of the enterprise is grouped into a department. For example, there may be production, finance and marketing departments in a manufacturing company, or underwriting and claims departments in an insurance company. A sales manager in this kind of department ation is responsible for the sale of all products manufactured by the company or a claims manager is responsible for dealing with all claims from different areas where the company operates. The relevant organization structure is functional organization. Advantages: 1. It is a simple form of grouping activities for organizations which manufacture only a limited number of products or render only a limited number of services. 2. It promotes excellence in performance because of development of expertise through specialization.. 3. It leads to improved planning and control of the key functions. 4. It ensures economy, for there is only one department related to one function for the entire company. Manpower and other resources of the company are effectively utilized by sharing them across products or projects. Drawbacks: 1 . I t fosters sub-goal loyalties. Each manager thinks only in terms of his own departmental goals and does not think in terms of the company as a whole. This results in inter-departmental conflicts and disagreements 2. It does not offer a good training ground for the overall development of a manager. The manager gains expertise in handling problems of his particular department only. 3. It is unsuitable where either geographical dispersal of units is required or emphasis on separate product lines is called for.
  • 17. MG2351 Principles of Management Department of ECE 2013 - 2014 2. Departmentation hased on products This form is suited for large organizations manufacturing a variety of products. Under this method, for each major product, a separate department is created and is put under the charge of a manager. He may also be made responsible for producing a given level of profit. Within each department, all the needed manufacturing, engineering, marketing, manpower and other facilities are created. Product departmentation is the logical pattern to follow when each product requires raw material, manufacturing technology, and marketing methods that are markedly different from those used by other products in the organisation. Examples are: Hindustan Unilever, Godrej, ITC, Reliance Industries, etc. The relevant organization structure is divisional organization. Advantages 1. This form relieves top management of operating task responsibility. It can therefore better concentrate on such centralised activities like finance, R & D and control. 2. This form enables top management to compare the performances of different products and invest more resources in profitable products and withdraw resources from unprofitable ones. 3. In this form, since the responsibility for each product's performance is entrusted to a particular departmental head, he is better stimulated to improve his performance. 4. In this form, those who work within a department derive greater satisfaction from identification with a recognisable goal. Drawbacks 1. This form results in duplication of staff and facilities. 2. Extra expenditure is incurred in maintaining a sales force for each product line. 3. Employment of a large number of managerial personnel is required. 4. Equipment in each product department may not be used fully. 3. Departmentation based on customers An enterprise may be divided into a number of departments on the basis of the customers that it services. For example, an electronics firm may have separate departments for military, industrial and consumer customers. A big automobile servicing enterprise may have separate departments for servicing cars, heavy vehicles and scooters, or an educational institution may have separate departments for day, evening and correspondence courses to impart education to full-time students, locally employed students and outstation students, respectively. One big advantage of this form is that it ensures full attention to major customer groups and this helps the company to earn goodwill. The drawbacks of this form are that (i) it may result in underutilisation of resources and facilities in some departments; and (ii) there may be duplication of facilities. Example is Airtel. The relevant organization structure is divisional organization. 4. Departmentation based on Regions or Territories When several production or marketing units of an organisation are geographically dispersed in various locations, it is logical to departmentalise those units on a geographical basis. The Indian Railways are departmentalised on this basis. Northern Railways, Western Railways, Southern Railways, Eastern Railways, Central Railways, etc. are departments in this sense. Here also the relevant organization structure is divisional organization. Advantages 1. It motivates each regional head to achieve high performance. 2. It provides each regional head an opportunity to adapt to his local situation and customer need with speed and accuracy. 3. It affords valuable top-management training and experience to middle-level executives. 4. It enables the organization to take advantage of locational factors, such as availability of raw materials, labour, market, etc. 5. It enables the organization to compare regional performances and invest more resources in profitable regions and withdraw resources from unprofitable ones.
  • 18. MG2351 Principles of Management Department of ECE 2013 - 2014 Drawbacks 1. It gives rise to duplication of various activities. Many routine and service functions performed by all the regional units can be performed centrally by the head office very economically.. 2. Various regional units may become so engrossed in short-run competition among themselves that they may forget the overall interest of the total organization. 6. Departmentation based on process Departmentation is here done on the basis of several discrete processes or technologies involved in the manufacture of a product. For example, a concern engaged in the production of vegetable oil may have separate departments for crushing, refining and finishing. Similarly a cotton textile mill may have separate departments for ginning, spinning, weaving, dyeing, printing, packing and sales. In this way, whenever work that would otherwise be done in several different locations in an enterprise is done in one place. Here also the relevant organization structure is divisional organization. Advantages 1. It facilitates the use of heavy and costly equipment in an efficient manner. There is very little chance for the equipment remaining idle or under-utilized because there is no duplication of the same. 2. It follows the principle of specialization— each department is engaged in doing a special type of work. This increases efficiency. 3. It is suitable for organizations which are engaged in the manufacture of products which involve a number of processes. Drawbacks 1. It does not provide good training ground and opportunity for the overall development of managerial talent. 2. When the process is sequential, the dependent departments generally become hostile to other departments and they express their resentment either by complaining directly about other departments or by passing on inferior work to their successive departments. 3. It is difficult to compare the performance of different process-based departments. 7. Combined Base Departmentation It is quite typical to find an organization following a different base of departmentation at different organisational levels. For example, an organization manufacturing agricultural machinery may follow 'product' as the base (tractor department, appliance department, generator department, etc) at the primary level (i.e., the level immediately below the chief executive), 'territory' as the base at the intermediate level and 'function' as the base at the ultimate level. Another form of combined base organization which is becoming very popular nowadays is matrix organization. In this form of organization (also called grid or lattice pattern), two types of departmentation—say, functional and product—may exist simultaneously. 7. Explain different methods of appraisal system. (Nov 2011) (i) Traditional methods • Graphic rating scales • Rankling method • Paired comparison method • Forced distribution method • Checklist methods • Essay form appraisal • Group appraisal
  • 19. MG2351 Principles of Management • Department of ECE 2013 - 2014 Confidential reports (ii) Modern methods • Behaviorally anchored rating scales • Assessment centres • Human – asset – accounting method • Management by objectives • 360º performance appraisal 8. What are the factors affecting organization structure? • Environment ( Mechanistic and organic system) • Strategy • Technology • People • Size of the firm. 9. What are the sources of recruitment? 1. Internal sources • Transfer and promotions • Job posting • Employee referrals II. External sources • Direct methods • Indirect methods • Third party methods 10. What are the factors affecting span of management? • Capacity of supervisor • Capacity of subordinate • Nature of work • Degree of decentralization • Degree of planning • Communication techniques • Use of staff assistance • Supervision from others. UNIT – IV Part A 1. Define Leading. It is a process of influencing people so that they will contribute to organization and group goals. 2. Define Motivation. (May 2008) It is a process of stimulating people to action to achieve/ accomplish desired goals. Motivation is defined as “those forces operating within the individual employee or subordinate which impel him to act or not to act in certain ways:. Motivation is mainly psychological. 3. Democratic leader. (Dec 2010)
  • 20. MG2351 Principles of Management Department of ECE 2013 - 2014 He consults with subordinates on proposed actions and decisions and encourages participation from them. .4. Define Job Enrichment. It attempts to make a job more varied by removing the dullness associated with performing repetitive operations 5. What is QWL? Increasing productivity and reducing inflation and as a way of obtaining industrial democracy and minimizing labor disputes. 6. Define Leadership. Art or process of influencing people so that they will strive willingly and enthusiastically towards achievement of the goal . 7. What are the Styles of Leadership ? • Autocratic leadership • Democratic leadership and • Free-rien leadership 8. Define Communication. Communication is the process of transmitting ideas, facts, opinions and feelings to others. Communication is a mutual interchange process that occurs between two or more persons. 9. What are the various types of communication? (Dec 2009) Formal,informal,vertical,horizontal,grapevine,written and oral communication. 10. What is Country club management? Mangers have little or no concern for production. but are concerned for people 11. What are the types of needs? • Primary Needs: Physiological requirements such as food, sleep, water and shelter • Secondary needs: Self esteem, status, affection, accomplishment and affiliation with others. 12. Explain - Maslow’s Needs. (May 2010) Physiological needs, Safety needs, Social needs, Self- Esteem needs and Self- ctualization needs. 13. Define Incentives An incentive is something which stimulates a person towards some determination 14. What are Job content factors? • Achievement, advancement, responsibility, • Respect, growth and development. 15. Define Expectancy theory. Force=Valance * Expectancy 16. What is an Equity theory? Outcomes by a person Outcomes by another person ------------------------------------ = ----------------------------------Inputs by a person inputs by another person 17. Define Creativity. Ability and power to develop new ideas 18. What is Innovation. Use of new ideas. 19. Define Brainstorming Improve problem solving by finding new ideas and unusual solution 20. What are the importance of leadership? Securing Co-operation, Creating confidence in the minds of employees, Providing good working climate.
  • 21. MG2351 Principles of Management Department of ECE 2013 - 2014 Part B 1. Write short notes on the managerial grid. A well-known approach to defining leadership styles is the managerial grid, developed some years ago by Robert Blake and Jane Mouton. Building on previous research that showed the importance of a manager's having concern both for production and for people, Blake and Mouton devised a clever device to dramatize this concern. This grid, shown in the figure, has been used throughout the world as a means of training managers and of identifying various combinations of leadership styles. Grid Dimensions: The grid has two dimensions: concern for people and concern for production. As Blake and Mouton have emphasized, the phrase "concern for" is meant to convey "how" managers are concerned about production or "how" they are concerned about people, and not such things as "how much" production they are concerned about getting out of a group. "Concern for production" includes the attitudes of a supervisor toward a wide variety of things, such as the quality of policy decisions, procedures and processes, creativeness of research, quality of staff services, work efficiency, and volume of output. "Concern for people" is likewise interpreted in a broad way. It includes such elements as degree of personal commitment toward goal achievement, maintenance of the self-esteem of workers, placement of responsibility on the basis of trust rather than obedience, provision of good working conditions, and maintenance of satisfying interpersonal relations. Four Extreme Styles: Blake and Mouton recognize four extremes of style. Under the 1.1 style (referred to as "impoverished management"), managers concern themselves very little with either people or production and have minimum involvement in their jobs; to all intents and purposes, they have abandoned their jobs and only mark time or act as messengers communicating information from superiors to subordinates. At the other extreme are the 9.9 managers, who display in their actions the highest possible dedication both to people and to production. They are real "team managers" who are able to mesh the production needs of the enterprise with the needs of individuals. Another style is 1.9 management (called "country club management" by some), in which managers have little or no concern for production but are concerned only for people. They promote an environment in which everyone is relaxed, friendly, and happy and no one is concerned about putting forth coordinated effort to accomplish enterprise goals. At another extreme are the 9.1 managers (sometimes referred to as "autocratic task managers") who are concerned only with developing an efficient operation, who have little or no concern for people, and who are quite autocratic in their style of leadership. By using these four extremes as points of reference, every managerial technique, approach, or style can be placed somewhere on the grid. Clearly, 5.5 managers have medium concern for production and for people. They do not set goals too high, and they are likely to have a rather benevolently autocratic attitude toward people. The managerial grid is a useful device of identifying and classifying managerial styles, but it does not tell us why a manager falls into one part or another of the grid. To determine the reason, one has to look at underlying causes, such as the personality characteristics of the leader or the followers, the ability and training of managers, the enterprise environment, and other situational factors that influence how leaders and followers act. 2. What are the steps in communication process? (May 2013) Communication has been defined as a process. The term process refers to identifiable flow of information through interrelated stages of analysis directed towards the achievement of an objective. The various elements of the communication process are sender, message, encoding, channel, receiver, decoding, understanding and feedback.
  • 22. MG2351 Principles of Management Department of ECE 2013 - 2014 Sender. Sender of the message is the person who intends to make contact with the objective of passing the message to other persons. In organisational context, sender may be a superior, a subordinate, a peer, or any other person. The organisational position of the sender determines the direction of flow of communication in an organisation. Message. This is the subject-matter of the communication which is intended to be passed to the receiver from the sender. The message may be in the form of thoughts, ideas, opinions, feelings, views, suggestions, orders, etc. Encoding. Since the subject-matter of communication, viz. thought, idea, etc., is abstract and intangible, its transmission requires the use of certain symbols such as words, pictures, gestures, etc. The process of converting the message into communication symbols is known as encoding. Channel. Message encoded into symbols is transmitted by the sender through a channel like written form, personal contact, phone call, e-mail, etc. depending on the two parties—the sender and receiver. Receiver. Receiver is the person to whom the symbols are transmitted. Like sender, he may be a superior, subordinate, peer, or any other person in the organisation. Decoding. Decoding is the reverse of encoding. The receiver receives the subject matter of communication in the form of communication symbols in which the sender has encoded his message. The receiver decodes these symbols into message. Understanding. Interpreting, obtaining meaning from the message communicated. Feedback. Feedback is necessary to ensure that the receiver has received the message and understood it in the same sense as the sender intended. The sender can never be sure whether or not his message has been effectively encoded, transmitted, received, decoded, and understood until it is confirmed by feedback. 3. Explain the motivation theories. (Nov 2012) 1. MASLOW’S HIERARCHY OF NEEDS THEORY One of the most widely mentioned theories of motivation is the hierarchy of needs theory put forth by psychologist Abraham Maslow. Maslow saw human needs in the form of a hierarchy, ascending from the lowest to the highest, and he concluded that when one set of needs is satisfied, the next higher level need gets activated. The basic human needs placed by Maslow in an ascending order of importance are: 1. Physiological Needs These are the basic needs for sustaining human life itself, such as food, water, warmth, shelter, and sleep. Maslow felt that until these needs are satisfied to the degree necessary to maintain life, other needs will not motivate people. 2. Security or Safety Needs These are the needs to be free of physical danger and of the fear of losing a job property, food, or shelter. 3. Affiliation or Social Needs Since people are social beings, they need to belong, to be accepted by others. It includes friendship, the need to love and be loved, socializing, etc. 4. Esteem Needs Once people begin to satisfy their need to belong, they tend to want to be held in esteem both by themselves and by others. This kind of need produces such satisfactions as respect, power, prestige, status, and self-confidence. 5. Self-actualizationNeeds This as the highest need in the hierarchy. It is the desire to become what one is capable of becoming—to fully realize one's potential and to accomplish what one is capable of achieving. Maslow suggests that the various levels are overlapping, each higher-level need emerging before the lower-level need has been completely satisfied. Since one need does not disappear when another
  • 23. MG2351 Principles of Management Department of ECE 2013 - 2014 emerges, all needs tend to be partially satisfied in each area. When the peak of a need is passed, that need ceases to be the primary motivator. The next level need then begins to dominate. 2. HERZBERG’S MOTIVATION-HYGIENE THEORY Maslow's need approach has been considerably modified by Frederick Herzberg. His research purports to find a two-factor theory of motivation. In one group of needs are such things as company policy and administration, supervision, working conditions, interpersonal relations, salary, status, and job security. These were found by Herzberg and his associates to be only dissatisfiers and not motivators. Their existence does not motivate in the sense of yielding satisfaction; their lack of existence would, however, result in dissatisfaction. Herzberg called them maintenance, hygiene or job context factors. In the second group, Herzberg listed certain satisfiers—and therefore motivators—all related to job content. They include achievement, recognition, challenging work, advancement, and growth in the job. Their existence will yield feelings of satisfaction but their absence will not lead to dissatisfaction. The satisfiers and dissatisfiers identified by Herzberg are similar to the factors suggested by Maslow: the dissatisfiers correspond to Maslow’s lower order needs (physiological, safety and social needs) while the satisfiers correspond to Maslow’s higher order needs (esteem and self-actualization needs). The first group of factors (the dissatisfiers) will not motivate in an organization; yet they must be present, or dissatisfaction will arise. The second group, or the job content factors, Herzberg found to be the real motivators because they have the potential of yielding a sense of satisfaction. 3. VROOM’S EXPECTANCY THEORY Another approach to motivation is the expectancy theory of the psychologist Victor H. Vroom. He holds that people will be motivated to do things to reach a goal if they believe in the worth of that goal and if they can see that what they do will help them in achieving it. Vroom's theory is that people's motivation toward doing anything will be determined by the value they place on the outcome of their effort (whether positive or negative), multiplied by the confidence they have that their efforts will materially aid in achieving a goal. In other words, Vroom makes the point that motivation is a product of the anticipated worth that an individual places on a goal and the chances he or she sees of achieving that goal. Using his own terms, Vroom's theory may be stated as Force = valence x expectancy Where force is the strength of a person motivation, valence is the strength of an individual's preference for an outcome, and expectancy is the probability that a particular action will lead to a desired outcome. When a person is indifferent about achieving a certain goal, a valence of zero occurs; there is a negative valence when the person would rather not achieve the goal. The result of either would be, of course, no motivation. Likewise, a person would have no motivation to achieve a goal if the expectancy were zero or negative. Hence the force exerted to do something will depend on both valence and expectancy. Moreover, a motive to accomplish some action might be determined by a desire to accomplish something else. Vroom identified these as first-level and second-level outcomes. For example, a person might be willing to work hard to please his boss (first-level outcome) in order to get higher pay (second-level outcome). Or a manager might be willing to work hard to achieve company goals (first-level outcome) for the sake of getting a promotion (secondlevel outcome). Vroom used the term ‘instrumentality’ to indicate the extent to which the achievement of first-level outcomes lead to the achievement of second-level outcomes. 4. PORTER AND LAWLER MODEL
  • 24. MG2351 Principles of Management Department of ECE 2013 - 2014 Lyman W. Porter and Edward E. Lawler III derived a substantially more complete model of motivation, built in large part on Vroom’s expectancy theory. In their study, they have applied this model primarily to managers. As the model indicates, the amount of effort (the strength of motivation and energy exerted) depends on the value of a reward plus the amount of energy a person believes is required and the probability of receiving the reward. The perceived effort and probability of actually getting a reward are, in turn, also influenced by the record of actual performance. Clearly, if people know they can do a job or if they haw done it before, they have a better appreciation of the effort required and know better the probability of rewards. Actual performance in a job (the doing of tasks or the meeting of goals) is determined principally by effort expended. But it is also greatly influenced by an individual's ability (knowledge and skills) to do the job and by his or her perception of what the required task is (the extent to which the person understands the goals, required activities, and other elements of a task). Performance, in turn, is seen as leading to intrinsic rewards (such as a sense accomplishment or self-actualization) and extrinsic rewards (such as working conditions and status). These rewards, tempered by what the individual sees as equitable, lead to satisfaction. But performance also influences sensed equitable rewards. Understanding what the individual sees as a fair reward for effort will necessarily affect the satisfaction derived. Likewise, the actual value of rewards will be influenced by satisfaction. 5. ADAMS’ EQUITY THEORY An important factor in motivation is whether individuals perceive the reward structure as being fair. One way of addressing this issue is through equity theory, which refers to an individual's subjective judgments about the fairness of the reward she or he got, relative to the inputs (which include many factors such as effort, experience, education, and so on), in comparison with the rewards of other. J. Stacy Adams has formulated the equity theory. The essential aspects of the equity theory may be shown as follows. Outcomes by a person = Outcomes by another person Inputs by a person Inputs by another person There should be a balance of the outcomes-inputs relationship for one person in comparison with that for another person. If people feel they are inequitably rewarded, they may be dissatisfied, reduce the quantity or quality of output, or leave the organization. They also can ask for a greater reward. If people perceive the rewards as equitable, they probably will continue at the same level of output. If people think the rewards are greater than what is considered equitable, they may work harder. It is also possible that some may discount the reward. The impact of inequity on a person is as follows: 1. Perceived inequity creates tension in the person. 2. The amount of tension is proportional to the magnitude of the inequity. 3. The tension created in the person will motivate him to reduce it. 4. The strength of the motivation to reduce inequity is proportional to the perceived inequity. 6. SKINNER’S REINFORCEMENT or BEHAVIOR MODIFICATION THEORY This theory has developed out of researches done by B.F. Skinner. According to this theory, people behave the way they do because, in past circumstances, they have learned that certain behaviors are associated with pleasant outcomes and certain other behaviours are associated with unpleasant outcomes. Because people generally prefer pleasant outcomes, they are likely to repeat behaviour that they have learned will have pleasant consequences. For example, people in an organisation are likely to obey a manager's instruction because they have learned at home and at school that obedience to authority leads to praise and disobedience leads to punishment. Thus, the frequency of
  • 25. MG2351 Principles of Management Department of ECE 2013 - 2014 the various kinds of behaviour in the organisation can be seen as contingent on the immediate consequences of these behaviours. The consequences that increase the frequency of a behaviour are positive reinforcement (e.g., praise or monetary reward) or negative reinforcement (i.e., escape from some aversive situation). An example of negative reinforcement might be a manager who requires all subordinates to attend an early morning staff meeting whenever performance of the organisation falls below a certain level. Subordinates would then work very hard to avoid the unpleasant early morning meetings and strive for a high level of performance. The consequences that decrease the frequency of a behaviour are extinction (i.e., ignoring the behaviour) and punishment (e.g. reprimand, fine, frown etc.). When reward or positive reinforcement is being used to increase the frequency of a desired behaviour the important point to be considered by the manager is the frequency of reward. Rewards can be administered continuously—every desired behaviour can be reinforced—or they can be administered on an intermittent basis. Normally a manager begins by reinforcing every desired behaviour. A secretary may be complemented every time he types an error-free letter. Then as the behaviour becomes permanent, the rewards are gradually tapered off to an intermittent or random basis. Instead of praising the secretary for every error-free letter, the manager may begin skipping now and then. 7. McCLELLAND'S NEEDS THEORY David C. McClelland has contributed to the understanding of motivation by identifying three types of basic motivating needs. He classified them as the need for power (n/PWR), need for affiliation (n/AFF), and need for achievement (n/ACH). Need for Power McClelland and other researchers have found that people with a high need for power have a great concern for exercising influence and control. Such individuals generally are seeking positions of leadership; they are frequently good conversationalists, though often argumentative; they are forceful, outspoken, hard-headed, and demanding; and they enjoy teaching and public speaking. Need for Affiliation People with a high need for affiliation usually derive pleasure from being loved and tend to avoid the pain of being rejected by a social group. As individuals, they are likely to be concerned with maintaining pleasant social relationships, to enjoy sense of intimacy and understanding, to be ready to console and help others in trouble, and to enjoy friendly interaction with others. Need for Achievement People with a high need for achievement have an intense desire for success and an equally intense fear of failure. They want to be challenged, and they set moderately difficult (but not impossible) goals for themselves. They take a realistic approach to risk; they are not likely to be gamblers but, rather, prefer to analyze and assess problems, assume personal responsibility for getting a job done, and like specific and prompt feedback on how they are doing. They tend to be restless, like to work long hours, do not worry unduly about failure if it does occur, and tend to like to run their own shows. The implication of this theory is that all three drives—power, affiliation, and achievement—are of relevance to management, since all must be recognized to make an organized enterprise work well. Because any organized enterprise and every department in it represents groups of individuals working together to achieve goals, organizations can improve the selection and placement processes if the needs of employees can be accurately assessed. 8. ALDERFER'S ERG THEORY
  • 26. MG2351 Principles of Management Department of ECE 2013 - 2014 Alderfer has provided an extension of the Maslow's need hierarchy and Herzberg's two-factor theory of motivation, particularly the former. Like the previous theories, Alderfer believes that there is a value in categorising needs and that there is a basic distinction between lower-order needs and higher-order needs. Based on the empirical evidences, he has found that there seems to be some overlapping between physiological, security and social needs. Also the lines of demarcation between social, esteem and achievement needs are not clear. Based on these observations, Alderfer has categorised the various needs into three categories; existence needs, relatedness needs, and growth needs. The first three letters of these needs are used to call it ERG theory. Existence Needs. Existence needs include all needs related to physiological and safety aspects of an individual. Thus, existence needs group physiological and safety needs of Maslow into one category as these have similar impact on the behavour of the individual. Relatedness Needs. Relatedness needs include all those needs that involve relationship with other people whom the individual cares. Relatedness needs cover Maslow's social needs and that part of esteem needs which is derived from the relationship with other people. Growth Needs. Growth needs involve the individual making creative efforts to achieve full potential in the existing environment. These include Maslow's self-actualisation need as well as that part of the esteem need which is internal to the individual like feeling of being unique, feeling of personal growth, etc. ERG theory offers the following propositions so far as satisfaction of various needs is concerned: 1. The three need categories form a hierarchy only in the sense of decreasing concreteness. As people move from a focus on existence to relatedness to growth needs, the ways in which they can satisfy those needs become increasingly abstract. 2. The rise in the level of satisfaction of any lower-order need may result in decrease in its importance. Its place is taken by another need. Thus, the individual is able to move to become productive and creative, and as he moves to this level, he sets a higher goal for himself. 3. People are likely to try to satisfy their most concrete needs first and then, they move on to the abstract needs. In this way, progression of need satisfaction of ERG theory is similar to Maslow's need hierarchy, that is, people first satisfy their lower needs and gradually progress to the satisfaction of higher needs, people can experience frustration-regression, that is, if people cannot satisfy their needs at a given level of abstraction, they 'drop back' and again focus on more concrete needs. 9. McGREGOR'S THEORY X AND THEORY Y The management's action of motivating human beings in the organisation, according to Douglas McGregor, involves certain assumptions, generalizations and hypotheses relating to human behaviour and human nature. They serve the purpose of predicting human behaviour. McGregor has characterised these assumptions in two opposite views, termed Theory X and Theory Y. Theory X. This is the traditional theory of human behaviour, In this theory, McGregor has certain assumptions about human behaviour. These assumptions are as follows: 1. Management is a process of directing employees’ efforts, motivating them, controlling their actions, modifying their behaviour to fit the needs of the organisation. 2. Without this active intervention by management, people would be passive— even resistant—to organisational needs. They must be persuaded, rewarded, punished, controlled, and their activities must be directed. 3. The average man is by nature indolent—he works as little as possible. 4. He lacks ambition, dislikes responsibility, prefers to be led. 5. He is inherently self-centered, indifferent to organisational needs. 6. He is, by nature, resistant to change. 7. He is gullible, not very bright, the ready dupe of the charlatan and the demagogue.
  • 27. MG2351 Principles of Management Department of ECE 2013 - 2014 These assumptions about human nature are negative in their approach, however much organisational processes have developed on these assumptions. Managers subscribing to these views about human nature attempt to structure, control and closely supervise their employees. They feel that external control is most appropriate for dealing with irresponsible and immature employees. Theory Y. The assumptions of Theory Y are described by McGregor in the following words: 1. The expenditure of physical and mental effort in work is as natural as play or rest. The average human being does not inherently dislike work. Depending upon controllable conditions, work may be a source of satisfaction or a source of punishment. 2. External control and the threat of punishment are not the only means for bringing about effort towards organisational objectives. Man will exercise self-direction and self-control in the service of objectives to which he is committed. 3. Commitment to objectives is a function of the reward associated with their achievement. The most significant of such awards, e.g. the satisfaction of ego and self-actualisation needs, can be a direct product of effort directed towards organisational objectives. 4. The average human being learns under proper conditions not only to accept, but to seek responsibility. Avoidance of responsibility, lack of ambition, and emphasis on security are generally consequences of experience, not inherent human characteristics. 5. The capacity to exercise a relatively high degree of imagination, ingenuity, and creativity in the solution of organisational problems is widely, not narrowly, distributed in the population. The assumptions of Theory Y suggest a new approach in management. It emphasises on the cooperative endeavour of management and employees. The attempt is to get maximum output with minimum amount of control and direction. Generally, no conflict is visible between organisational goals and individual goals. Thus, the attempts of employees which are in their best interests are also in the interests of the organisation. 4. Explain the various types of leadership styles. • The autocratic leader commands and expects compliance, is dogmatic and positive, and leads by the ability to withhold or give rewards and punishment. • The democratic, or participative leader consults with subordinates on proposed actions and decisions and encourages participation from them. This type of leader ranges from the person who does not take action without subordinates' concurrence to the one who makes decisions but consults with subordinates before doing so. • The free-rein leader uses his or her power very little, if at all, giving subordinates a high degree of independence in their operations. Such leaders depend largely on subordinates to set their own goals and the means of achieving them, and they see their role as one of aiding the operation of followers by furnishing them with information and acting primarily as a contact with the group's external environment. 5. What are the barriers in communication? (May 2013) A. Semantic barriers: 1. Words/symbols with different meanings. Communication symbols usually have a variety of meanings. For example, the English word ‘round’ has 110 different meanings. The 500 most common English words have an average of 28 definitions each. Moreover, a particular word may have different meanings in different languages. Similarly, non-verbal symbols may convey different meanings to different persons. In such situations, the receiver may interpret differently and communication breaks down.
  • 28. MG2351 Principles of Management Department of ECE 2013 - 2014 2. Badly expressed messages. Poorly chosen or empty words/phrases, careless omission, lack of coherence, bad organization of ideas, awkward sentence structure, inadequate vocabulary are some common faults that may distort a message. 3. Faulty translations. The message must be put into words appropriate to the framework in which the receiver operates. This requires a high level of linguistic capability. Approximate understanding of words and their faulty translation lead to poor communication. 4. Different backgrounds. One reason for distortion of meaning is that different individuals often interpret the same communication differently; each individual uses his own frame of reference. This frame of reference is based on particular experience and knowledge. Thus, when people with different knowledge and experiences try to communicate, they often have trouble getting their meanings across. 5. In-group language (Jargon). Often, occupational or social groups develop their own terminology or in-group language. This special language, though providing a means for precise and quick communication within the group, creates severe communication breakdown when outsiders or other groups are involved. B. Psychological barriers: 6. Premature evaluation: It is the tendency of prematurely judging communication instead of receiving it with an open mind. This may cause misunderstanding in the mind of the receiver as well as de-motivate the sender. 6. Inattention. Another common barrier is that many receivers simply do not pay attention to the message. One reason people do not pay attention is selective listening. Selective listening results from a common tendency to block out information that conflicts with what we believe. When we listen to a speech or read a newspaper, we generally pay attention only to those things that confirm our beliefs. Sometimes people do not pay attention to communication because they are victims of communication overload or because the information is unsolicited. 7. Poor retention. Studies show that employees retain only 50 per cent of communicated information. 8. Distrust, threat or fear of communicator. Distrust, threat and fear undermine communication. In a climate containing these forces, any message will be viewed with skepticism. Distrust can be due to inconsistent behavior by the superior. Faced with threats – real or imagined – people tend to tighten up, become defensive, and distort information. C. Organizational barriers: 9. Faulty organization. In a large-scale enterprise where the chain of command is too long or the span of control too big, communication will be poor. This is because successive transmissions of the same message are decreasingly accurate. In oral communication, around 30 percent of the information is lost in each transmission. 10. Filtering: Filtering refers to intentionally withholding or deliberately manipulating information by the sender, either because the sender believes that the receiver does not need all the information or that the receiver is better off not knowing all aspects of the situation. It could also be that the receiver is simply told what he wants to hear. 11. Organizational policy: If the policy is not supportive to the flow of communication in different directions, communication flow would not be smooth and adequate.
  • 29. MG2351 Principles of Management Department of ECE 2013 - 2014 12. Organizational rules and regulations: These affect the flow of communication by prescribing the subject-matters to be communicated and also the channel through which these are to be communicated. 13. Status relationships: The placing of people in superior/subordinate capacity in the formal organization structure blocks the flow of communication, more so in the upward direction. 6. Explain trait approaches to leadership. Trait is defined as a relatively enduring quality of an individual. The trait approach seeks to determine ‘what makes a successful leader’ from the leader’s personal characteristics. Prior to 1950, studies of leadership were based largely on an attempt to identify the traits that leaders possess. Starting with the "great man" theory that leaders are born and not made, a belief dating back to the ancient Greeks and Romans, researchers have tried to identify the physical, mental, and personality traits of various leaders. The major traits identified by researchers in the course of their leadership studies include the following: physical factors such as height, weight, physique, energy, health, appearance; mental qualities such as intelligence, alertness, administrative ability; personality characteristics such as self-confidence, initiative, persistence, ambition, cheerfulness, enthusiasm, decisiveness; and social characteristics such as sociability, adaptability, dominance, aggressiveness, etc. These various traits can be classified as innate and acquired traits. Innate traits are those which are possessed by individuals since their birth. These traits are natural and God-gifted. Acquired traits are those which can be acquired or increased through learning, imitating, training, and other such processes. 7. Explain various methods of communication. (Nov 2012) Communication can be classified into different types: • Formal communication • Informal communication • Downward communication • Upward communication • Horizontal communication • Diagonal communication • Oral communication • Written communication • Electronic communication, and • Non-verbal communication. 8. List out the various communication networks in an organization. I. Formal channel • Single chain • Wheel • Circular • Free flow • Inverted V • Single channel Vs multiple channels II Informal chain or grapevine
  • 30. MG2351 Principles of Management Department of ECE 2013 - 2014 9. What are the computer – based communications in the modern business era? • Electronic mail • Internet and extranet links • Videoconferencing (i) One – way video and two – way audio (ii) Two – way video and audio 10. Explain the factors determining effective supervision. • Leadership • Closeness of supervision • Employee – orientation or human relations • Group cohesiveness • Delegation • Other factors UNIT - V Part A 1. What is control? (May 2010) Control is the measurement and correction of performance in order to make sure the enterprise objectives and the plans devised to attain them are accomplished.. 2. What is concurrent control? (May 2011) This control measures for taking corrective action while any programme meet any obstacle in this activities. 3. State the difference between feedback and feed forward control technique? S. No. 1. Feedback Feed forward It measures only the output of the process It measures the input of the process 2. It is submissive approach It is aggressive approach 3. Less benefit More benefit 4. Define Budget? (Dec 2011) A Budget the expression of a firm’s plan is financial form for a period of time in to the future. 5. Define budgetary control? (May 2010) A system which uses budgets as a means of planning and controlling all aspects of producing and selling commodities and services. 6. Write the objectives of budgetary control? • It aims at maximization of profits • To plan and control the income and expenditure of the organization • To provide adequate working capital 7. What are classification of budget? • Functional classification- Sales, production, cash, capital and master budget • Time classification- Short, current and long term budget • Activity level- Fixed and flexible budget 8. What is zero base budget? (Dec 2009)
  • 31. MG2351 Principles of Management Department of ECE 2013 - 2014 Initially the budget is designed from a zero base. The main element is ZBB is future objective orientation. 9. What are the difference between PERT and CPM? S. No. CPM PERT 1. It is activity oriented PERT is event oriented 2. CPM is planning device It is control device 3. It estimates only one time It estimates three times 4. It is a deterministic model It is probabilistic model 10. Define MIS. (Dec 2011) MIS is more advance technology for solving its basic requirements. MIS used for decision making in the various functional areas of business. 11. Define productivity. Productivity is a measure of how much input is required to produce a given output. 12. Define OR. OR is a systematic analysis of a problem through scientific methods, carried out by appropriate specialists, working together as a team, finding an optimum and the most appropriate solution to meet the given objective under a given set of constraints. 13. Define Linear Programming? It is a mathematical technique in operation research and a plan of action solve a given problem involving linearly related variables in order to achieve the laid down objectives in the form of minimizing or maximizing the objective function under given set of constraints. 14. What is Inventory Control? Inventory control refers to the control of raw materials and purchased material in store and regulation of investment in them. 15. What is JIT? (May 2007) Just in time inventory system, in this method the suppliers delivers the materials to the production spot just in time to be assembled. This method reduces the cost of iventory. 16. What are objectives of value engineering? a) It is a special type of cost reduction technique. b) Modify and improve product design c) Reduce the product cost d) Increase the profit e) Simplify the product 17. What is MNC? (May 2008) Multinational corporation is an enterprise which own or control production or service facilities outside the country in which they are based. 18. Write some advantages of MNCs? (Dec 2009) • It can promote quality product at low price • MNC leads to increase in production aggregate employment, exports and imports of the required inputs • It increases the government revenues. 19. What is work simplification? It is the process of obtaining the participation of workers in simplifying their work through time study, motion study, work flow analysis and layout of work situation.
  • 32. MG2351 Principles of Management Department of ECE 2013 - 2014 20. Define quality circles? Quality circles are groups of people, from the same organizational area, who meet regularly to solve problems they experience at work. Members are trained in solving problems, in applying statistical quality control and working in groups. Part B 1. What is the importance of controlling? • Policy verification • Adjustments in operations • Measuring performance • Comparing actual with standards • Finding out deviations • Correction of deviation 2. List out the critical types of critical point standards. • Physical standards • Cost standards • Capital standards • Revenue standards • Program standards • Intangible standards • Goals as standards • Strategic control. 3. What are the various budgetary control techniques? (May 2013) I. Traditional techniques • Personal observation • Break – even analysis • Statistical reports • Budgetary control II. Modern techniques • Management audit • Return on investment • PERT and CPM • Management information systems 4. What are the factors affecting control? • Technology • Human resources
  • 33. MG2351 Principles of Management • • • • • • • • Department of ECE Government policy Machinery and equipment Skill of the worker Materials Plant equipment Land and buildings Capital Research and development. 5. Explain the role of operations research in business and management. I. Production management • Allocation of resources • Project scheduling • Inventory policy • Equipment replacement and maintenance II. Finance management • Fund flow analysis • Credit policies • Capital requirement III. Purchase and procurement • Rules for purchasing • Determining the quantity IV. Distribution • Location of warehouses • Side of the warehouses • Transportation strategies V. Marketing management • • • Product selection Competitive strategies Advertising strategies. 6. What are the steps involved in planning and control? • Routing • Scheduling • Dispatching • Inspection • Follow – up. 7. What is the importance of financial statements? 2013 - 2014
  • 34. MG2351 Principles of Management • • • • • • Department of ECE 2013 - 2014 Management Shareholders Creditors Labors Public Government 8. What are the steps involved in standard purchasing procedure? (N0v 2012) • Processing the requisition • Location and choice of suppliers • Placing of orders • Follow up and expediting • Invoice and clearance • Maintenance of records. 9. Explain the various methods of purchasing. • Purchasing according to the requirement • Price forecasting method • Purchasing for some definite future period • Market purchasing • Speculative purchasing • Contract purchasing • Scheduled purchasing • Public buying • Tender purchasing. 10. What are the steps in quality control? • Fixing the quality standards • Evaluation of measurement of quality • Comparing the measured quality with the standard quality • Finding out the deviation • Reasons for variation • Taking corrective action. ***********************************